Programs & Examples On #Event routing

Angular2 change detection: ngOnChanges not firing for nested object

suppose you have a nested object, like

var obj = {"parent": {"child": {....}}}

If you passed the reference of the complete object, like

[wholeObj] = "obj"

In that case, you can't detect the changes in the child objects, so to overcome this problem you can also pass the reference of the child object through another property, like

[wholeObj] = "obj" [childObj] = "obj.parent.child"

So you can also detect the changes from the child objects too.

ngOnChanges(changes: SimpleChanges) { 
    if (changes.childObj) {// your logic here}
}

Where to place JavaScript in an HTML file?

The answer is depends how you are using the objects of javascript. As already pointed loading the javascript files at footer rather than header certainly improves the performance but care should be taken that the objects which are used are initialized later than they are loaded at footer. One more way is load the 'js' files placed in folder which will be available to all the files.

Using CMake with GNU Make: How can I see the exact commands?

cmake --build . --verbose

On Linux and with Makefile generation, this is likely just calling make VERBOSE=1 under the hood, but cmake --build can be more portable for your build system, e.g. working across OSes or if you decide to do e.g. Ninja builds later on:

mkdir build
cd build
cmake ..
cmake --build . --verbose

Its documentation also suggests that it is equivalent to VERBOSE=1:

--verbose, -v

Enable verbose output - if supported - including the build commands to be executed.

This option can be omitted if VERBOSE environment variable or CMAKE_VERBOSE_MAKEFILE cached variable is set.

Why does the order in which libraries are linked sometimes cause errors in GCC?

You may can use -Xlinker option.

g++ -o foobar  -Xlinker -start-group  -Xlinker libA.a -Xlinker libB.a -Xlinker libC.a  -Xlinker -end-group 

is ALMOST equal to

g++ -o foobar  -Xlinker -start-group  -Xlinker libC.a -Xlinker libB.a -Xlinker libA.a  -Xlinker -end-group 

Careful !

  1. The order within a group is important ! Here's an example: a debug library has a debug routine, but the non-debug library has a weak version of the same. You must put the debug library FIRST in the group or you will resolve to the non-debug version.
  2. You need to precede each library in the group list with -Xlinker

#1055 - Expression of SELECT list is not in GROUP BY clause and contains nonaggregated column this is incompatible with sql_mode=only_full_group_by

Solution 1: Remove ONLY_FULL_GROUP_BY from mysql console

mysql > SET GLOBAL sql_mode=(SELECT REPLACE(@@sql_mode,'ONLY_FULL_GROUP_BY',''));

you can read more here

Solution 2: Remove ONLY_FULL_GROUP_BY from phpmyadmin

Open phpmyadmin & select localhost
Click on menu Variables & scroll down for sql mode
Click on edit button to change the values & remove ONLY_FULL_GROUP_BY & click on save. 

click here to see image

Add and remove attribute with jquery

If you want to do this, you need to save it in a variable first. So you don't need to use id to query this element every time.

var el = $("#page_navigation1");

$("#add").click(function(){
  el.attr("id","page_navigation1");
});     

$("#remove").click(function(){
  el.removeAttr("id");
});     

form action with javascript

Absolutely valid.

    <form action="javascript:alert('Hello there, I am being submitted');">
        <button type="submit">
            Let's do it
        </button>
    </form>
    <!-- Tested in Firefox, Chrome, Edge and Safari -->

So for a short answer: yes, this is an option, and a nice one. It says "when submitted, please don't go anywhere, just run this script" - quite to the point.

A minor improvement

To let the event handler know which form we're dealing with, it would seem an obvious way to pass on the sender object:

    <form action="javascript:myFunction(this)">  <!-- should work, but it won't -->

But instead, it will give you undefined. You can't access it because javascript: links live in a separate scope. Therefore I'd suggest the following format, it's only 13 characters more and works like a charm:

    <form action="javascript:;" onsubmit="myFunction(this)">  <!-- now you have it! -->

... now you can access the sender form properly. (You can write a simple "#" as action, it's quite common - but it has a side effect of scrolling to the top when submitting.)

Again, I like this approach because it's effortless and self-explaining. No "return false", no jQuery/domReady, no heavy weapons. It just does what it seems to do. Surely other methods work too, but for me, this is The Way Of The Samurai.

A note on validation

Forms only get submitted if their onsubmit event handler returns something truthy, so you can easily run some preemptive checks:

    <form action="/something.php" onsubmit="return isMyFormValid(this)">

Now isMyFormValid will run first, and if it returns false, server won't even be bothered. Needless to say, you will have to validate on server side too, and that's the more important one. But for quick and convenient early detection this is fine.

support FragmentPagerAdapter holds reference to old fragments

I solved the problem by saving the fragments in SparceArray:

public abstract class SaveFragmentsPagerAdapter extends FragmentPagerAdapter {

    SparseArray<Fragment> fragments = new SparseArray<>();

    public SaveFragmentsPagerAdapter(FragmentManager fm) {
        super(fm);
    }

    @Override
    public Object instantiateItem(ViewGroup container, int position) {
        Fragment fragment = (Fragment) super.instantiateItem(container, position);
        fragments.append(position, fragment);
        return fragment;
    }

    @Nullable
    public Fragment getFragmentByPosition(int position){
        return fragments.get(position);
    }

}

Android Overriding onBackPressed()

Override the onBackPressed() method as per the example by codeMagic, and remove the call to super.onBackPressed(); if you do not want the default action (finishing the current activity) to be executed.

How to check whether a string is Base64 encoded or not

var base64Rejex = /^(?:[A-Z0-9+\/]{4})*(?:[A-Z0-9+\/]{2}==|[A-Z0-9+\/]{3}=|[A-Z0-9+\/]{4})$/i;
var isBase64Valid = base64Rejex.test(base64Data); // base64Data is the base64 string

if (isBase64Valid) {
    // true if base64 formate
    console.log('It is base64');
} else {
    // false if not in base64 formate
    console.log('it is not in base64');
}

How can I determine installed SQL Server instances and their versions?

One more option would be to run SQLSERVER discovery report..go to installation media of sqlserver and double click setup.exe

enter image description here

and in the next screen,go to tools and click discovery report as shown below

enter image description here

This will show you all the instances present along with entire features..below is a snapshot on my pc enter image description here

How to read an excel file in C# without using Microsoft.Office.Interop.Excel libraries

You can try OleDB to read data from excel file. Please try as follow..

DataSet ds_Data = new DataSet();
OleDbConnection oleCon = new OleDbConnection();

string strExcelFile = @"C:\Test.xlsx";
oleCon.ConnectionString = "Provider=Microsoft.ACE.OLEDB.12.0;Data Source=" + strExcelFile + ";Extended Properties=\"Excel 12.0;IMEX=1;HDR=NO;TypeGuessRows=0;ImportMixedTypes=Text\"";;

 string SpreadSheetName = "";

OleDbDataAdapter Adapter = new OleDbDataAdapter();
OleDbConnection conn = new OleDbConnection(sConnectionString);

string strQuery;
conn.Open();

int workSheetNumber = 0;

DataTable ExcelSheets = conn.GetOleDbSchemaTable(System.Data.OleDb.OleDbSchemaGuid.Tables, new object[] { null, null, null, "TABLE" });

SpreadSheetName = ExcelSheets.Rows[workSheetNumber]["TABLE_NAME"].ToString();

strQuery = "select * from [" + SpreadSheetName + "] ";
OleDbCommand cmd = new OleDbCommand(strQuery, conn);
Adapter.SelectCommand = cmd;
DataSet dsExcel = new DataSet();
Adapter.Fill(dsExcel);
conn.Close();

how to get file path from sd card in android

There are different Names of SD-Cards.

This Code check every possible Name (I don't guarantee that these are all names but the most are included)

It prefers the main storage.

 private String SDPath() {
    String sdcardpath = "";

    //Datas
    if (new File("/data/sdext4/").exists() && new File("/data/sdext4/").canRead()){
        sdcardpath = "/data/sdext4/";
    }
    if (new File("/data/sdext3/").exists() && new File("/data/sdext3/").canRead()){
        sdcardpath = "/data/sdext3/";
    }
    if (new File("/data/sdext2/").exists() && new File("/data/sdext2/").canRead()){
        sdcardpath = "/data/sdext2/";
    }
    if (new File("/data/sdext1/").exists() && new File("/data/sdext1/").canRead()){
        sdcardpath = "/data/sdext1/";
    }
    if (new File("/data/sdext/").exists() && new File("/data/sdext/").canRead()){
        sdcardpath = "/data/sdext/";
    }

    //MNTS

    if (new File("mnt/sdcard/external_sd/").exists() && new File("mnt/sdcard/external_sd/").canRead()){
        sdcardpath = "mnt/sdcard/external_sd/";
    }
    if (new File("mnt/extsdcard/").exists() && new File("mnt/extsdcard/").canRead()){
        sdcardpath = "mnt/extsdcard/";
    }
    if (new File("mnt/external_sd/").exists() && new File("mnt/external_sd/").canRead()){
        sdcardpath = "mnt/external_sd/";
    }
    if (new File("mnt/emmc/").exists() && new File("mnt/emmc/").canRead()){
        sdcardpath = "mnt/emmc/";
    }
    if (new File("mnt/sdcard0/").exists() && new File("mnt/sdcard0/").canRead()){
        sdcardpath = "mnt/sdcard0/";
    }
    if (new File("mnt/sdcard1/").exists() && new File("mnt/sdcard1/").canRead()){
        sdcardpath = "mnt/sdcard1/";
    }
    if (new File("mnt/sdcard/").exists() && new File("mnt/sdcard/").canRead()){
        sdcardpath = "mnt/sdcard/";
    }

    //Storages
    if (new File("/storage/removable/sdcard1/").exists() && new File("/storage/removable/sdcard1/").canRead()){
        sdcardpath = "/storage/removable/sdcard1/";
    }
    if (new File("/storage/external_SD/").exists() && new File("/storage/external_SD/").canRead()){
        sdcardpath = "/storage/external_SD/";
    }
    if (new File("/storage/ext_sd/").exists() && new File("/storage/ext_sd/").canRead()){
        sdcardpath = "/storage/ext_sd/";
    }
    if (new File("/storage/sdcard1/").exists() && new File("/storage/sdcard1/").canRead()){
        sdcardpath = "/storage/sdcard1/";
    }
    if (new File("/storage/sdcard0/").exists() && new File("/storage/sdcard0/").canRead()){
        sdcardpath = "/storage/sdcard0/";
    }
    if (new File("/storage/sdcard/").exists() && new File("/storage/sdcard/").canRead()){
        sdcardpath = "/storage/sdcard/";
    }
    if (sdcardpath.contentEquals("")){
        sdcardpath = Environment.getExternalStorageDirectory().getAbsolutePath();
    }

    Log.v("SDFinder","Path: " + sdcardpath);
    return sdcardpath;
}

Using a PHP variable in a text input value = statement

You need, for example:

<input type="text" name="idtest" value="<?php echo $idtest; ?>" />

The echo function is what actually outputs the value of the variable.

How do I check if an integer is even or odd?

// C#
bool isEven = ((i % 2) == 0);

Convert SVG to image (JPEG, PNG, etc.) in the browser

change svg to match your element

function svg2img(){
    var svg = document.querySelector('svg');
    var xml = new XMLSerializer().serializeToString(svg);
    var svg64 = btoa(xml); //for utf8: btoa(unescape(encodeURIComponent(xml)))
    var b64start = 'data:image/svg+xml;base64,';
    var image64 = b64start + svg64;
    return image64;
};svg2img()

Hibernate Auto Increment ID

@Id
@GeneratedValue(strategy=GenerationType.AUTO)
private int id;

and you leave it null (0) when persisting. (null if you use the Integer / Long wrappers)

In some cases the AUTO strategy is resolved to SEQUENCE rathen than to IDENTITY or TABLE, so you might want to manually set it to IDENTITY or TABLE (depending on the underlying database).

It seems SEQUENCE + specifying the sequence name worked for you.

How can I convert a long to int in Java?

Shortest, most safe and easiest solution is:

long myValue=...;
int asInt = Long.valueOf(myValue).intValue();

Do note, the behavior of Long.valueOf is as such:

Using this code:

System.out.println("Long max: " + Long.MAX_VALUE);
System.out.println("Int max: " + Integer.MAX_VALUE);        
long maxIntValue = Integer.MAX_VALUE;
System.out.println("Long maxIntValue to int: " + Long.valueOf(maxIntValue).intValue());
long maxIntValuePlusOne = Integer.MAX_VALUE + 1;
System.out.println("Long maxIntValuePlusOne to int: " + Long.valueOf(maxIntValuePlusOne).intValue());
System.out.println("Long max to int: " + Long.valueOf(Long.MAX_VALUE).intValue());

Results into:

Long max: 9223372036854775807
Int max: 2147483647
Long max to int: -1
Long maxIntValue to int: 2147483647
Long maxIntValuePlusOne to int: -2147483648

Code for a simple JavaScript countdown timer?

You can do as follows with pure JS. You just need to provide the function with the number of seconds and it will do the rest.

_x000D_
_x000D_
var insertZero = n => n < 10 ? "0"+n : ""+n,_x000D_
   displayTime = n => n ? time.textContent = insertZero(~~(n/3600)%3600) + ":" +_x000D_
                                             insertZero(~~(n/60)%60) + ":" +_x000D_
                                             insertZero(n%60)_x000D_
                        : time.textContent = "IGNITION..!",_x000D_
 countDownFrom = n => (displayTime(n), setTimeout(_ => n ? sid = countDownFrom(--n)_x000D_
                                                         : displayTime(n), 1000)),_x000D_
           sid;_x000D_
countDownFrom(3610);_x000D_
setTimeout(_ => clearTimeout(sid),20005);
_x000D_
<div id="time"></div>
_x000D_
_x000D_
_x000D_

Cheap way to search a large text file for a string

If there is no way to tell where the string will be (first half, second half, etc) then there is really no optimized way to do the search other than the builtin "find" function. You could reduce the I/O time and memory consumption by not reading the file all in one shot, but at 4kb blocks (which is usually the size of an hard disk block). This will not make the search faster, unless the string is in the first part of the file, but in all case will reduce memory consumption which might be a good idea if the file is huge.

how to convert `content://media/external/images/media/Y` to `file:///storage/sdcard0/Pictures/X.jpg` in android?

Will something like this work for you? What this does is query the content resolver to find the file path data that is stored for that content entry

public static String getRealPathFromUri(Context context, Uri contentUri) {
    Cursor cursor = null;
    try {
        String[] proj = { MediaStore.Images.Media.DATA };
        cursor = context.getContentResolver().query(contentUri, proj, null, null, null);
        int column_index = cursor.getColumnIndexOrThrow(MediaStore.Images.Media.DATA);
        cursor.moveToFirst();
        return cursor.getString(column_index);
    } finally {
        if (cursor != null) {
            cursor.close();
        }
    }
}

This will end up giving you an absolute file path that you can construct a file uri from

Error in Eclipse: "The project cannot be built until build path errors are resolved"

  1. Go to Project > Properties > Java Compiler > Building
  2. Look under Build Path Problems
  3. Un-check "Abort build when build path error occurs"
    It won't solve all your errors but at least it will let you run your program :)

How to automatically generate a stacktrace when my program crashes

ulimit -c <value> sets the core file size limit on unix. By default, the core file size limit is 0. You can see your ulimit values with ulimit -a.

also, if you run your program from within gdb, it will halt your program on "segmentation violations" (SIGSEGV, generally when you accessed a piece of memory that you hadn't allocated) or you can set breakpoints.

ddd and nemiver are front-ends for gdb which make working with it much easier for the novice.

How to execute an oracle stored procedure?

Both 'is' and 'as' are valid syntax. Output is disabled by default. Try a procedure that also enables output...

create or replace procedure temp_proc is
begin
  DBMS_OUTPUT.ENABLE(1000000);
  DBMS_OUTPUT.PUT_LINE('Test');
end;

...and call it in a PLSQL block...

begin
  temp_proc;
end;

...as SQL is non-procedural.

Encrypt Password in Configuration Files?

If you are using java 8 the use of the internal Base64 encoder and decoder can be avoided by replacing

return new BASE64Encoder().encode(bytes);

with

return Base64.getEncoder().encodeToString(bytes);

and

return new BASE64Decoder().decodeBuffer(property);

with

return Base64.getDecoder().decode(property);

Note that this solution doesn't protect your data as the methods for decrypting are stored in the same place. It just makes it more difficult to break. Mainly it avoids to print it and show it to everybody by mistake.

How can I make the contents of a fixed element scrollable only when it exceeds the height of the viewport?

Add this to your code for fixed height and add one scroll.

.fixedbox {
    max-height: auto;
    overflow-y: scroll;
}

Resolving LNK4098: defaultlib 'MSVCRT' conflicts with

IMO this link from Yochai Timmer was very good and relevant but painful to read. I wrote a summary.

Yochai, if you ever read this, please see the note at the end.


For the original post read : warning LNK4098: defaultlib "LIBCD" conflicts with use of other libs

Error

LINK : warning LNK4098: defaultlib "LIBCD" conflicts with use of other libs; use /NODEFAULTLIB:library

Meaning

one part of the system was compiled to use a single threaded standard (libc) library with debug information (libcd) which is statically linked

while another part of the system was compiled to use a multi-threaded standard library without debug information which resides in a DLL and uses dynamic linking

How to resolve

  • Ignore the warning, after all it is only a warning. However, your program now contains multiple instances of the same functions.

  • Use the linker option /NODEFAULTLIB:lib. This is not a complete solution, even if you can get your program to link this way you are ignoring a warning sign: the code has been compiled for different environments, some of your code may be compiled for a single threaded model while other code is multi-threaded.

  • [...] trawl through all your libraries and ensure they have the correct link settings

In the latter, as it in mentioned in the original post, two common problems can arise :

  • You have a third party library which is linked differently to your application.

  • You have other directives embedded in your code: normally this is the MFC. If any modules in your system link against MFC all your modules must nominally link against the same version of MFC.

For those cases, ensure you understand the problem and decide among the solutions.


Note : I wanted to include that summary of Yochai Timmer's link into his own answer but since some people have trouble to review edits properly I had to write it in a separate answer. Sorry

Limit Get-ChildItem recursion depth

This is a function that outputs one line per item, with indentation according to depth level. It is probably much more readable.

function GetDirs($path = $pwd, [Byte]$ToDepth = 255, [Byte]$CurrentDepth = 0)
{
    $CurrentDepth++
    If ($CurrentDepth -le $ToDepth) {
        foreach ($item in Get-ChildItem $path)
        {
            if (Test-Path $item.FullName -PathType Container)
            {
                "." * $CurrentDepth + $item.FullName
                GetDirs $item.FullName -ToDepth $ToDepth -CurrentDepth $CurrentDepth
            }
        }
    }
}

It is based on a blog post, Practical PowerShell: Pruning File Trees and Extending Cmdlets.

How to declare a local variable in Razor?

you can put everything in a block and easily write any code that you wish in that block just exactly the below code :

@{
        bool isUserConnected = string.IsNullOrEmpty(Model.CreatorFullName);
        if (isUserConnected)
        { // meaning that the viewing user has not been saved
            <div>
                <div> click to join us </div>
                <a id="login" href="javascript:void(0);" style="display: inline; ">join</a>
            </div>
        }
    }

it helps you to have at first a cleaner code and also you can prevent your page from loading many times different blocks of codes

What's the best strategy for unit-testing database-driven applications?

I've actually used your first approach with quite some success, but in a slightly different ways that I think would solve some of your problems:

  1. Keep the entire schema and scripts for creating it in source control so that anyone can create the current database schema after a check out. In addition, keep sample data in data files that get loaded by part of the build process. As you discover data that causes errors, add it to your sample data to check that errors don't re-emerge.

  2. Use a continuous integration server to build the database schema, load the sample data, and run tests. This is how we keep our test database in sync (rebuilding it at every test run). Though this requires that the CI server have access and ownership of its own dedicated database instance, I say that having our db schema built 3 times a day has dramatically helped find errors that probably would not have been found till just before delivery (if not later). I can't say that I rebuild the schema before every commit. Does anybody? With this approach you won't have to (well maybe we should, but its not a big deal if someone forgets).

  3. For my group, user input is done at the application level (not db) so this is tested via standard unit tests.

Loading Production Database Copy:
This was the approach that was used at my last job. It was a huge pain cause of a couple of issues:

  1. The copy would get out of date from the production version
  2. Changes would be made to the copy's schema and wouldn't get propagated to the production systems. At this point we'd have diverging schemas. Not fun.

Mocking Database Server:
We also do this at my current job. After every commit we execute unit tests against the application code that have mock db accessors injected. Then three times a day we execute the full db build described above. I definitely recommend both approaches.

How do you implement a re-try-catch?

https://github.com/tusharmndr/retry-function-wrapper/tree/master/src/main/java/io

int MAX_RETRY = 3; 
RetryUtil.<Boolean>retry(MAX_RETRY,() -> {
    //Function to retry
    return true;
});

Array initialization syntax when not in a declaration

I can't answer the why part.

But if you want something dynamic then why don't you consider Collection ArrayList.

ArrrayList can be of any Object type.

And if as an compulsion you want it as an array you can use the toArray() method on it.

For example:

            ArrayList<String> al = new ArrayList<String>();
            al.add("one");
            al.add("two");
            String[] strArray = (String[]) al.toArray(new String[0]);

I hope this might help you.

What is ToString("N0") format?

It is a sort of format specifier for formatting numeric results. There are additional specifiers on the link.

What N does is that it separates numbers into thousand decimal places according to your CultureInfo and represents only 2 decimal digits in floating part as is N2 by rounding right-most digit if necessary.

N0 does not represent any decimal place but rounding is applied to it.

Let's exemplify.

using System;
using System.Globalization;


namespace ConsoleApp1
{
    class Program
    {
        static void Main(string[] args)
        {
            double x = 567892.98789;
            CultureInfo someCulture = new CultureInfo("da-DK", false);

            // 10 means left-padded = right-alignment
            Console.WriteLine(String.Format(someCulture, "{0:N} denmark", x));
            Console.WriteLine("{0,10:N} us", x); 

            // watch out rounding 567,893
            Console.WriteLine(String.Format(someCulture, "{0,10:N0}", x)); 
            Console.WriteLine("{0,10:N0}", x);

            Console.WriteLine(String.Format(someCulture, "{0,10:N5}", x));
            Console.WriteLine("{0,10:N5}", x);


            Console.ReadKey();

        }
    }
}

It yields,

567.892,99 denmark
567,892.99 us
   567.893
   567,893
567.892,98789
567,892.98789

How to solve error "Missing `secret_key_base` for 'production' environment" (Rails 4.1)

While you can use initializers like the other answers, the conventional Rails 4.1+ way is to use the config/secrets.yml. The reason for the Rails team to introduce this is beyond the scope of this answer but the TL;DR is that secret_token.rb conflates configuration and code as well as being a security risk since the token is checked into source control history and the only system that needs to know the production secret token is the production infrastructure.

You should add this file to .gitignore much like you wouldn't add config/database.yml to source control either.

Referencing Heroku's own code for setting up config/database.yml from DATABASE_URL in their Buildpack for Ruby, I ended up forking their repo and modified it to create config/secrets.yml from SECRETS_KEY_BASE environment variable.

Since this feature was introduced in Rails 4.1, I felt it was appropriate to edit ./lib/language_pack/rails41.rb and add this functionality.

The following is the snippet from the modified buildpack I created at my company:

class LanguagePack::Rails41 < LanguagePack::Rails4

  # ...

  def compile
    instrument "rails41.compile" do
      super
      allow_git do
        create_secrets_yml
      end
    end
  end

  # ...

  # writes ERB based secrets.yml for Rails 4.1+
  def create_secrets_yml
    instrument 'ruby.create_secrets_yml' do
      log("create_secrets_yml") do
        return unless File.directory?("config")
        topic("Writing config/secrets.yml to read from SECRET_KEY_BASE")
        File.open("config/secrets.yml", "w") do |file|
          file.puts <<-SECRETS_YML
<%
raise "No RACK_ENV or RAILS_ENV found" unless ENV["RAILS_ENV"] || ENV["RACK_ENV"]
%>

<%= ENV["RAILS_ENV"] || ENV["RACK_ENV"] %>:
  secret_key_base: <%= ENV["SECRET_KEY_BASE"] %>
          SECRETS_YML
        end
      end
    end
  end

  # ...

end

You can of course extend this code to add other secrets (e.g. third party API keys, etc.) to be read off of your environment variable:

...
<%= ENV["RAILS_ENV"] || ENV["RACK_ENV"] %>:
  secret_key_base: <%= ENV["SECRET_KEY_BASE"] %>
  third_party_api_key: <%= ENV["THIRD_PARTY_API"] %>

This way, you can access this secret in a very standard way:

Rails.application.secrets.third_party_api_key

Before redeploying your app, be sure to set your environment variable first: Setting SECRET_KEY_BASE in Heroku Dashboard

Then add your modified buildpack (or you're more than welcome to link to mine) to your Heroku app (see Heroku's documentation) and redeploy your app.

The buildpack will automatically create your config/secrets.yml from your environment variable as part of the dyno build process every time you git push to Heroku.

EDIT: Heroku's own documentation suggests creating config/secrets.yml to read from the environment variable but this implies you should check this file into source control. In my case, this doesn't work well since I have hardcoded secrets for development and testing environments that I'd rather not check in.

NSPhotoLibraryUsageDescription key must be present in Info.plist to use camera roll

You need to paste these two in your info.plist, The only way that worked in iOS 11 for me.

    <key>NSPhotoLibraryUsageDescription</key>
    <string>This app requires access to the photo library.</string>

    <key>NSPhotoLibraryAddUsageDescription</key>
    <string>This app requires access to the photo library.</string>

Java - How Can I Write My ArrayList to a file, and Read (load) that file to the original ArrayList?

In Java 8 you can use Files.write() method with two arguments: Path and List<String>, something like this:

List<String> clubNames = clubs.stream()
    .map(Club::getName)
    .collect(Collectors.toList())

try {
    Files.write(Paths.get(fileName), clubNames);
} catch (IOException e) {
    log.error("Unable to write out names", e);
}

How to style icon color, size, and shadow of Font Awesome Icons

You can also just add style inline:

<i class="icon-ok-sign" style="color:green"></i>

<i class="icon-warning-sign" style="color:red"></i>

How to remove specific substrings from a set of strings in Python?

Update for Python 3.9

In python 3.9 you could remove suffix using str.removesuffix('suffix')

From the docs,

If the string ends with the suffix string and that suffix is not empty, return string[:-len(suffix)]. Otherwise, return a copy of the original string:

set1  = {'Apple.good','Orange.good','Pear.bad','Pear.good','Banana.bad','Potato.bad'}

set2 = set()

for s in set1:
   set2.add(s.removesuffix(".good").removesuffix(".bad"))

or using set comprehension:

set2 = {s.removesuffix(".good").removesuffix(".bad") for s in set1}
   
print(set2)


Output:
{'Orange', 'Pear', 'Apple', 'Banana', 'Potato'}

How to encode text to base64 in python

1) This works without imports in Python 2:

>>>
>>> 'Some text'.encode('base64')
'U29tZSB0ZXh0\n'
>>>
>>> 'U29tZSB0ZXh0\n'.decode('base64')
'Some text'
>>>
>>> 'U29tZSB0ZXh0'.decode('base64')
'Some text'
>>>

(although this doesn't work in Python3 )

2) In Python 3 you'd have to import base64 and do base64.b64decode('...') - will work in Python 2 too.

How do I specify "close existing connections" in sql script

Go to management studio and do everything you describe, only instead of clicking OK, click on Script. It will show the code it will run which you can then incorporate in your scripts.

In this case, you want:

ALTER DATABASE [MyDatabase] SET SINGLE_USER WITH ROLLBACK IMMEDIATE
GO

Keep getting No 'Access-Control-Allow-Origin' error with XMLHttpRequest

Remove:

httpRequest.setRequestHeader( 'Access-Control-Allow-Origin', '*');

... and add:

httpRequest.withCredentials = false;

Compare two Timestamp in java

From : http://download.oracle.com/javase/6/docs/api/java/sql/Timestamp.html#compareTo(java.sql.Timestamp)

public int compareTo(Timestamp ts)

Compares this Timestamp object to the given Timestamp object. Parameters: ts - the Timestamp object to be compared to this Timestamp object Returns: the value 0 if the two Timestamp objects are equal; a value less than 0 if this Timestamp object is before the given argument; and a value greater than 0 if this Timestamp object is after the given argument. Since: 1.4

font size in html code

you dont need those quotes

<td style="padding-left: 5px;padding-bottom:3px; font-size: 35px;"> <b>Datum:</b><br/>
                        November 2010 </td>

How can I view the shared preferences file using Android Studio?

You could simply create a special Activity for debugging purpose:

@SuppressWarnings("unchecked")
public void loadPreferences() {
// create a textview with id (tv_pref) in Layout.
TextView prefTextView;
prefTextView = (TextView) findViewById(R.id.tv_pref);
    Map<String, ?> prefs = PreferenceManager.getDefaultSharedPreferences(
            context).getAll();
    for (String key : prefs.keySet()) {
        Object pref = prefs.get(key);
        String printVal = "";
        if (pref instanceof Boolean) {
            printVal =  key + " : " + (Boolean) pref;
        }
        if (pref instanceof Float) {
            printVal =  key + " : " + (Float) pref;
        }
        if (pref instanceof Integer) {
            printVal =  key + " : " + (Integer) pref;
        }
        if (pref instanceof Long) {
            printVal =  key + " : " + (Long) pref;
        }
        if (pref instanceof String) {
            printVal =  key + " : " + (String) pref;
        }
        if (pref instanceof Set<?>) {
            printVal =  key + " : " + (Set<String>) pref;
        }
        // Every new preference goes to a new line
        prefTextView.append(printVal + "\n\n");     
    }
}
// call loadPreferences() in the onCreate of your Activity.

font-family is inherit. How to find out the font-family in chrome developer pane?

The inherit value, when used, means that the value of the property is set to the value of the same property of the parent element. For the root element (in HTML documents, for the html element) there is no parent element; by definition, the value used is the initial value of the property. The initial value is defined for each property in CSS specifications.

The font-family property is special in the sense that the initial value is not fixed in the specification but defined to be browser-dependent. This means that the browser’s default font family is used. This value can be set by the user.

If there is a continuous chain of elements (in the sense of parent-child relationships) from the root element to the current element, all with font-family set to inherit or not set at all in any style sheet (which also causes inheritance), then the font is the browser default.

This is rather uninteresting, though. If you don’t set fonts at all, browsers defaults will be used. Your real problem might be different – you seem to be looking at the part of style sheets that constitute a browser style sheet. There are probably other, more interesting style sheets that affect the situation.

Why am I getting an Exception with the message "Invalid setup on a non-virtual (overridable in VB) member..."?

You'll get this error as well if you are verifying that an extension method of an interface is called.

For example if you are mocking:

var mockValidator = new Mock<IValidator<Foo>>();
mockValidator
  .Verify(validator => validator.ValidateAndThrow(foo, null));

You will get the same exception because .ValidateAndThrow() is an extension on the IValidator<T> interface.

public static void ValidateAndThrow<T>(this IValidator<T> validator, T instance, string ruleSet = null)...

Moment Js UTC to Local Time

To convert UTC to local time

let UTC = moment.utc()
let local = moment(UTC).local()

Or you want directly get the local time

let local = moment()

_x000D_
_x000D_
var UTC = moment.utc()_x000D_
console.log(UTC.format()); // UTC time_x000D_
_x000D_
var cLocal = UTC.local()_x000D_
console.log(cLocal.format()); // Convert UTC time_x000D_
_x000D_
var local = moment();_x000D_
console.log(local.format()); // Local time
_x000D_
<script src="https://cdnjs.cloudflare.com/ajax/libs/moment.js/2.22.2/moment.min.js"></script>
_x000D_
_x000D_
_x000D_

Error:Execution failed for task ':app:transformClassesWithDexForDebug' in android studio

Check Whether multidex enabled or not in your build.gradle(app level) under dependencies.if not place like below

dependecies{
multidexEnabled true
}

Check your gradle.properties(app level).if you see the below code

#org.gradle.jvmargs=-Xmx2048m -XX:MaxPermSize=512m XX:+HeapDumpOnOutOfMemoryError -Dfile.encoding=UTF-8

remove # before the line ,then it should be like this

org.gradle.jvmargs=-Xmx2048m -XX:MaxPermSize=512m XX:+HeapDumpOnOutOfMemoryError -Dfile.encoding=UTF-8

Does Arduino use C or C++?

Both are supported. To quote the Arduino homepage,

The core libraries are written in C and C++ and compiled using avr-gcc

Note that C++ is a superset of C (well, almost), and thus can often look very similar. I am not an expert, but I guess that most of what you will program for the Arduino in your first year on that platform will not need anything but plain C.

Modify XML existing content in C#

The XmlTextWriter is usually used for generating (not updating) XML content. When you load the xml file into an XmlDocument, you don't need a separate writer.

Just update the node you have selected and .Save() that XmlDocument.

Code Sign error: The identity 'iPhone Developer' doesn't match any valid certificate/private key pair in the default keychain

I had the same problem. I'm very new to iphone development and it was my first time trying to load my program onto my iphone. The message is correct, you need to create a certificate in the keychain. The best walkthrough is here:

http://developer.apple.com/ios/manage/overview/index.action

You of course need to have a developer account (need to have paid the $100 yearly fee).

I hope this helps.

How to solve Object reference not set to an instance of an object.?

You need to initialize the list first:

protected List<string> list = new List<string>();

How to set up default schema name in JPA configuration?

For others who use spring-boot, java based configuration,

I set the schema value in application.properties

spring.jpa.properties.hibernate.dialect=...
spring.jpa.properties.hibernate.default_schema=...

If...Then...Else with multiple statements after Then

Multiple statements are to be separated by a new line:

If SkyIsBlue Then
  StartEngines
  Pollute
ElseIf SkyIsRed Then
  StopAttack
  Vent
ElseIf SkyIsYellow Then
  If Sunset Then
    Sleep
  ElseIf Sunrise or IsMorning Then
    Smoke
    GetCoffee
  Else
    Error
  End If
Else
  Joke
  Laugh
End If

Indentation Error in Python

In Notepad++

View --->Show Symbols --->Show White Spaces and Tabs(select)

replace all tabs with spaces.

What is REST? Slightly confused

http://en.wikipedia.org/wiki/Representational_State_Transfer

The basic idea is that instead of having an ongoing connection to the server, you make a request, get some data, show that to a user, but maybe not all of it, and then when the user does something which calls for more data, or to pass some up to the server, the client initiates a change to a new state.

Looking for a 'cmake clean' command to clear up CMake output

I used zsxwing's answer successfully to solve the following problem:

I have source that I build on multiple hosts (on a Raspberry Pi Linux board, on a VMware Linux virtual machine, etc.)

I have a Bash script that creates temporary directories based on the hostname of the machine like this:

# Get hostname to use as part of directory names
HOST_NAME=`uname -n`

# Create a temporary directory for cmake files so they don't
# end up all mixed up with the source.

TMP_DIR="cmake.tmp.$HOSTNAME"

if [ ! -e $TMP_DIR ] ; then
  echo "Creating directory for cmake tmp files : $TMP_DIR"
  mkdir $TMP_DIR
else
  echo "Reusing cmake tmp dir : $TMP_DIR"
fi

# Create makefiles with CMake
#
# Note: switch to the temporary dir and build parent 
#       which is a way of making cmake tmp files stay
#       out of the way.
#
# Note 2: to clean up cmake files, it is OK to
#        "rm -rf" the temporary directories

echo
echo Creating Makefiles with cmake ...

cd $TMP_DIR

cmake ..

# Run makefile (in temporary directory)

echo
echo Starting build ...

make

Wavy shape with css

Here's another way to do it :) The concept is to create a clip-path polygon with the wave as one side.

This approach is fairly flexible. You can change the position (left, right, top or bottom) in which the wave appears, change the wave function to any function(t) which maps to [0,1]). The polygon can also be used for shape-outside, which lets text flow around the wave when in 'left' or 'right' orientation.

At the end, an example you can uncomment which demonstrates animating the wave.

_x000D_
_x000D_
 _x000D_
_x000D_
function PolyCalc(f /*a function(t)  from [0, infinity) => [0, 1]*/, _x000D_
                  s, /*a slice function(y, i) from y [0,1] => [0, 1], with slice index, i, in [0, n]*/_x000D_
         w /*window size in seconds*/,_x000D_
                  n /*sample size*/,_x000D_
                  o /*orientation => left/right/top/bottom - the 'flat edge' of the polygon*/ _x000D_
                  ) _x000D_
{_x000D_
 this.polyStart = "polygon(";_x000D_
  this.polyLeft = this.polyStart + "0% 0%, "; //starts in the top left corner_x000D_
  this.polyRight = this.polyStart + "100% 0%, "; //starts in the top right corner_x000D_
  this.polyTop = this.polyStart + "0% 0%, "; // starts in the top left corner_x000D_
  this.polyBottom = this.polyStart + "0% 100%, ";//starts in the bottom left corner_x000D_
  _x000D_
  var self = this;_x000D_
  self.mapFunc = s;_x000D_
  this.func = f;_x000D_
  this.window = w;_x000D_
  this.count = n;_x000D_
  var dt = w/n;  _x000D_
_x000D_
  switch(o) {_x000D_
    case "top":_x000D_
      this.poly = this.polyTop; break;_x000D_
    case "bottom":_x000D_
      this.poly = this.polyBottom; break;_x000D_
   case "right":_x000D_
     this.poly = this.polyRight; break;_x000D_
   case "left":_x000D_
   default:_x000D_
    this.poly = this.polyLeft; break;_x000D_
    }_x000D_
    _x000D_
  this.CalcPolygon = function(t) {_x000D_
   var p = this.poly;_x000D_
    for (i = 0; i < this.count; i++) {_x000D_
      x = 100 * i/(this.count-1.0);_x000D_
      y = this.func(t + i*dt);_x000D_
      if (typeof self.mapFunc !== 'undefined')_x000D_
       y=self.mapFunc(y, i);_x000D_
      y*=100;_x000D_
      switch(o) {_x000D_
        case "top": _x000D_
          p += x + "% " + y + "%, "; break;_x000D_
        case "bottom":_x000D_
          p += x + "% " + (100-y) + "%, "; break;_x000D_
       case "right":_x000D_
         p += (100-y) + "% " + x + "%, "; break;_x000D_
       case "left":_x000D_
        default:_x000D_
         p += y + "% " + x + "%, "; break;          _x000D_
      }_x000D_
    }_x000D_
    _x000D_
    switch(o) { _x000D_
      case "top":_x000D_
        p += "100% 0%)"; break;_x000D_
      case "bottom":_x000D_
        p += "100% 100%)";_x000D_
        break;_x000D_
     case "right":_x000D_
       p += "100% 100%)"; break;_x000D_
     case "left":_x000D_
      default:_x000D_
       p += "0% 100%)"; break;_x000D_
    }_x000D_
    _x000D_
    return p;_x000D_
  }_x000D_
};_x000D_
_x000D_
var text = document.querySelector("#text");_x000D_
var divs = document.querySelectorAll(".wave");_x000D_
var freq=2*Math.PI; //angular frequency in radians/sec_x000D_
var windowWidth = 1; //the time domain window which determines the range from [t, t+windowWidth] that will be evaluated to create the polygon_x000D_
var sampleSize = 60;_x000D_
divs.forEach(function(wave) {_x000D_
  var loc = wave.classList[1];_x000D_
_x000D_
  var polyCalc = new PolyCalc(_x000D_
   function(t) { //The time domain wave function_x000D_
     return (Math.sin(freq * t) + 1)/2; //sine is [-1, -1], so we remap to [0,1]_x000D_
    },_x000D_
    function(y, i) { //slice function, takes the time domain result and the slice index and returns a new value in [0, 1]  _x000D_
      return MapRange(y, 0.0, 1.0, 0.65, 1.0);  //Here we adjust the range of the wave to 'flatten' it out a bit.  We don't use the index in this case, since it is irrelevant_x000D_
    },_x000D_
    windowWidth, //1 second, which with an angular frequency of 2pi rads/sec will produce one full period._x000D_
    sampleSize, //the number of samples to make, the larger the number, the smoother the curve, but the more pionts in the final polygon_x000D_
    loc //the location_x000D_
  );_x000D_
  _x000D_
    var polyText = polyCalc.CalcPolygon(0);_x000D_
    wave.style.clipPath = polyText;_x000D_
    wave.style.shapeOutside = polyText;_x000D_
    wave.addEventListener("click",function(e) {document.querySelector("#polygon").innerText = polyText;});_x000D_
  });_x000D_
_x000D_
function MapRange(value, min, max, newMin, newMax) {_x000D_
  return value * (newMax - newMin)/(max-min) + newMin;_x000D_
}_x000D_
_x000D_
//Animation - animate the wave by uncommenting this section_x000D_
//Also demonstrates a slice function which uses the index of the slice to alter the output for a dampening effect._x000D_
/*_x000D_
var t = 0;_x000D_
var speed = 1/180;_x000D_
_x000D_
var polyTop = document.querySelector(".top");_x000D_
_x000D_
var polyTopCalc = new PolyCalc(_x000D_
   function(t) {_x000D_
     return (Math.sin(freq * t) + 1)/2;_x000D_
    },_x000D_
    function(y, i) {       _x000D_
      return MapRange(y, 0.0, 1.0, (sampleSize-i)/sampleSize, 1.0);_x000D_
    },_x000D_
    windowWidth, sampleSize, "top"_x000D_
  );_x000D_
_x000D_
function animate() {_x000D_
  var polyT = polyTopCalc.CalcPolygon(t);    _x000D_
    t+= speed;_x000D_
    polyTop.style.clipPath = polyT;    _x000D_
    requestAnimationFrame(animate);_x000D_
}_x000D_
_x000D_
requestAnimationFrame(animate);_x000D_
*/
_x000D_
div div {_x000D_
  padding:10px;_x000D_
  /*overflow:scroll;*/_x000D_
}_x000D_
_x000D_
.left {_x000D_
  height:100%;_x000D_
  width:35%;_x000D_
  float:left;_x000D_
}_x000D_
_x000D_
.right {_x000D_
  height:200px;_x000D_
  width:35%;_x000D_
  float:right;_x000D_
}_x000D_
_x000D_
.top { _x000D_
  width:100%;_x000D_
  height: 200px;  _x000D_
}_x000D_
_x000D_
.bottom {_x000D_
  width:100%;_x000D_
  height:200px;_x000D_
}_x000D_
_x000D_
.green {_x000D_
  background:linear-gradient(to bottom, #b4ddb4 0%,#83c783 17%,#52b152 33%,#008a00 67%,#005700 83%,#002400 100%); _x000D_
} _x000D_
_x000D_
.mainContainer {_x000D_
  width:100%;_x000D_
  float:left;_x000D_
}_x000D_
_x000D_
#polygon {_x000D_
  padding-left:20px;_x000D_
  margin-left:20px;_x000D_
  width:100%;_x000D_
}
_x000D_
<div class="mainContainer">_x000D_
_x000D_
  <div class="wave top green">_x000D_
    Click to see the polygon CSS_x000D_
  </div>_x000D_
  _x000D_
  <!--div class="wave left green">_x000D_
  </div-->_x000D_
  <!--div class="wave right green">_x000D_
  </div-->  _x000D_
  <!--div class="wave bottom green"></div-->  _x000D_
</div>_x000D_
<div id="polygon"></div>
_x000D_
_x000D_
_x000D_

Join/Where with LINQ and Lambda

I've done something like this;

var certificationClass = _db.INDIVIDUALLICENSEs
    .Join(_db.INDLICENSECLAsses,
        IL => IL.LICENSE_CLASS,
        ILC => ILC.NAME,
        (IL, ILC) => new { INDIVIDUALLICENSE = IL, INDLICENSECLAsse = ILC })
    .Where(o => 
        o.INDIVIDUALLICENSE.GLOBALENTITYID == "ABC" &&
        o.INDIVIDUALLICENSE.LICENSE_TYPE == "ABC")
    .Select(t => new
        {
            value = t.PSP_INDLICENSECLAsse.ID,
            name = t.PSP_INDIVIDUALLICENSE.LICENSE_CLASS,                
        })
    .OrderBy(x => x.name);

What is a Question Mark "?" and Colon ":" Operator Used for?

Also just though I'd post the answer to another related question I had,

a = x ? : y;

Is equivalent to:

a = x ? x : y;

If x is false or null then the value of y is taken.

Eloquent Collection: Counting and Detect Empty

When using ->get() you cannot simply use any of the below:

if (empty($result)) { }
if (!$result) { }
if ($result) { }

Because if you dd($result); you'll notice an instance of Illuminate\Support\Collection is always returned, even when there are no results. Essentially what you're checking is $a = new stdClass; if ($a) { ... } which will always return true.

To determine if there are any results you can do any of the following:

if ($result->first()) { } 
if (!$result->isEmpty()) { }
if ($result->count()) { }
if (count($result)) { }

You could also use ->first() instead of ->get() on the query builder which will return an instance of the first found model, or null otherwise. This is useful if you need or are expecting only one result from the database.

$result = Model::where(...)->first();
if ($result) { ... }

Notes / References

Bonus Information

The Collection and the Query Builder differences can be a bit confusing to newcomers of Laravel because the method names are often the same between the two. For that reason it can be confusing to know what one you’re working on. The Query Builder essentially builds a query until you call a method where it will execute the query and hit the database (e.g. when you call certain methods like ->all() ->first() ->lists() and others). Those methods also exist on the Collection object, which can get returned from the Query Builder if there are multiple results. If you're not sure what class you're actually working with, try doing var_dump(User::all()) and experimenting to see what classes it's actually returning (with help of get_class(...)). I highly recommend you check out the source code for the Collection class, it's pretty simple. Then check out the Query Builder and see the similarities in function names and find out when it actually hits the database.

Why do multiple-table joins produce duplicate rows?

This might sound like a really basic "DUH" answer, but make sure that the column you're using to Lookup from on the merging file is actually full of unique values!

I noticed earlier today that PowerQuery won't throw you an error (like in PowerPivot) and will happily allow you to run a Many-Many merge. This will result in multiple rows being produced for each record that matches with a non-unique value.

what is Promotional and Feature graphic in Android Market/Play Store?

It's here http://www.android.com/market/featured.html Weirdly, you don't get to that page if you start from the android market and hit "featured". Mary

View more than one project/solution in Visual Studio

Just right click on the Visual Studio icon and then select "New Window" from the contextual toolbar that appears on the bottom in Windows 8. A new instance of Visual Studio will launch and then you can open your second project.

How to export data to an excel file using PHPExcel

I currently use this function in my project after a series of googling to download excel file from sql statement

    // $sql = sql query e.g "select * from mytablename"
    // $filename = name of the file to download 
        function queryToExcel($sql, $fileName = 'name.xlsx') {
                // initialise excel column name
                // currently limited to queries with less than 27 columns
        $columnArray = array("A", "B", "C", "D", "E", "F", "G", "H", "I", "J", "K", "L", "M", "N", "O", "P", "Q", "R", "S", "T", "U", "V", "W", "X", "Y", "Z");
                // Execute the database query
                $result =  mysql_query($sql) or die(mysql_error());

                // Instantiate a new PHPExcel object
                $objPHPExcel = new PHPExcel();
                // Set the active Excel worksheet to sheet 0
                $objPHPExcel->setActiveSheetIndex(0);
                // Initialise the Excel row number
                $rowCount = 1;
    // fetch result set column information
                $finfo = mysqli_fetch_fields($result);
// initialise columnlenght counter                
$columnlenght = 0;
                foreach ($finfo as $val) {
// set column header values                   
  $objPHPExcel->getActiveSheet()->SetCellValue($columnArray[$columnlenght++] . $rowCount, $val->name);
                }
// make the column headers bold
                $objPHPExcel->getActiveSheet()->getStyle($columnArray[0]."1:".$columnArray[$columnlenght]."1")->getFont()->setBold(true);

                $rowCount++;
                // Iterate through each result from the SQL query in turn
                // We fetch each database result row into $row in turn

                while ($row = mysqli_fetch_array($result, MYSQL_NUM)) {
                    for ($i = 0; $i < $columnLenght; $i++) {
                        $objPHPExcel->getActiveSheet()->SetCellValue($columnArray[$i] . $rowCount, $row[$i]);
                    }
                    $rowCount++;
                }
// set header information to force download
                header('Content-type: application/vnd.ms-excel');
                header('Content-Disposition: attachment; filename="' . $fileName . '"');
                // Instantiate a Writer to create an OfficeOpenXML Excel .xlsx file        
                // Write the Excel file to filename some_excel_file.xlsx in the current directory                
                $objWriter = new PHPExcel_Writer_Excel2007($objPHPExcel); 
                // Write the Excel file to filename some_excel_file.xlsx in the current directory
                $objWriter->save('php://output');
            }

How to create an 2D ArrayList in java?

I want to create a 2D array that each cell is an ArrayList!

If you want to create a 2D array of ArrayList.Then you can do this :

ArrayList[][] table = new ArrayList[10][10];
table[0][0] = new ArrayList(); // add another ArrayList object to [0,0]
table[0][0].add(); // add object to that ArrayList

jQuery selector for id starts with specific text

If all your divs start with editDialog as you stated, then you can use the following selector:

$("div[id^='editDialog']")

Or you could use a class selector instead if it's easier for you

<div id="editDialog-0" class="editDialog">...</div>

$(".editDialog")

Can I run Keras model on gpu?

See if your script is running GPU in Task manager. If not, suspect your CUDA version is right one for the tensorflow version you are using, as the other answers suggested already.

Additionally, a proper CUDA DNN library for the CUDA version is required to run GPU with tensorflow. Download/extract it from here and put the DLL (e.g., cudnn64_7.dll) into CUDA bin folder (e.g., C:\Program Files\NVIDIA GPU Computing Toolkit\CUDA\v10.1\bin).

A JRE or JDK must be available in order to run Eclipse. No JVM was found after searching the following locations

Change the vm value in eclipse.ini file with the correct path to your JDK something like this,

-vm /Library/Java/JavaVirtualMachines/jdk-11.0.5.jdk/Contents/Home/bin

Path to eclipse.ini looks to me something like this,

/Users/tomcat/eclipse/jee-2018-09/Eclipse.app/Contents/Eclipse

How do I run a Python program?

Python itself comes with an editor that you can access from the IDLE File > New File menu option.

Write the code in that file, save it as [filename].py and then (in that same file editor window) press F5 to execute the code you created in the IDLE Shell window.

Note: it's just been the easiest and most straightforward way for me so far.

What is the equivalent of bigint in C#?

For most of the cases it is long(int64) in c#

How to install Jdk in centos

Here is something that might help. Use the root privileges. if you have .bin then simply add the execution permission to the bin file.

chmod a+x jdk*.bin

next step is to run the .bin file which is simply

./jdk*.bin in the location you want to install.

you are done.

How to join two JavaScript Objects, without using JQUERY

1)

var merged = {};
for(key in obj1)
    merged[key] = obj1[key];
for(key in obj2)
    merged[key] = obj2[key];

2)

var merged = {};
Object.keys(obj1).forEach(k => merged[k] = obj1[k]);
Object.keys(obj2).forEach(k => merged[k] = obj2[k]);

OR

Object.keys(obj1)
    .concat(Object.keys(obj2))
    .forEach(k => merged[k] = k in obj2 ? obj2[k] : obj1[k]);

3) Simplest way:

var merged = {};
Object.assign(merged, obj1, obj2);

Activate a virtualenv with a Python script

The top answer only works for Python 2.x

For Python 3.x, use this:

activate_this_file = "/path/to/virtualenv/bin/activate_this.py"

exec(compile(open(activate_this_file, "rb").read(), activate_this_file, 'exec'), dict(__file__=activate_this_file))

Reference: What is an alternative to execfile in Python 3?

How to store Query Result in variable using mysql

Surround that select with parentheses.

SET @v1 := (SELECT COUNT(*) FROM user_rating);
SELECT @v1;

Create a custom View by inflating a layout?

A bit old, but I thought sharing how I'd do it, based on chubbsondubs' answer: I use FrameLayout (see Documentation), since it is used to contain a single view, and inflate into it the view from the xml.

Code following:

public class MyView extends FrameLayout {
    public MyView(Context context, AttributeSet attrs, int defStyle) {
        super(context, attrs, defStyle);
        initView();
    }

    public MyView(Context context, AttributeSet attrs) {
        super(context, attrs);
        initView();
    }

    public MyView(Context context) {
        super(context);
        initView();
    }

    private void initView() {
        inflate(getContext(), R.layout.my_view_layout, this);
    }
}

Get the week start date and week end date from week number

Let us break the problem down to two parts:

1) Determine the day of week

The DATEPART(dw, ...) returns a number, 1...7, relative to DATEFIRST setting (docs). The following table summarizes the possible values:

                                                   @@DATEFIRST
+------------------------------------+-----+-----+-----+-----+-----+-----+-----+-----+
|                                    |  1  |  2  |  3  |  4  |  5  |  6  |  7  | DOW |
+------------------------------------+-----+-----+-----+-----+-----+-----+-----+-----+
|  DATEPART(dw, /*Mon*/ '20010101')  |  1  |  7  |  6  |  5  |  4  |  3  |  2  |  1  |
|  DATEPART(dw, /*Tue*/ '20010102')  |  2  |  1  |  7  |  6  |  5  |  4  |  3  |  2  |
|  DATEPART(dw, /*Wed*/ '20010103')  |  3  |  2  |  1  |  7  |  6  |  5  |  4  |  3  |
|  DATEPART(dw, /*Thu*/ '20010104')  |  4  |  3  |  2  |  1  |  7  |  6  |  5  |  4  |
|  DATEPART(dw, /*Fri*/ '20010105')  |  5  |  4  |  3  |  2  |  1  |  7  |  6  |  5  |
|  DATEPART(dw, /*Sat*/ '20010106')  |  6  |  5  |  4  |  3  |  2  |  1  |  7  |  6  |
|  DATEPART(dw, /*Sun*/ '20010107')  |  7  |  6  |  5  |  4  |  3  |  2  |  1  |  7  |
+------------------------------------+-----+-----+-----+-----+-----+-----+-----+-----+

The last column contains the ideal day-of-week value for Monday to Sunday weeks*. By just looking at the chart we come up with the following equation:

(@@DATEFIRST + DATEPART(dw, SomeDate) - 1 - 1) % 7 + 1

2) Calculate the Monday and Sunday for given date

This is trivial thanks to the day-of-week value. Here is an example:

WITH TestData(SomeDate) AS (
    SELECT CAST('20001225' AS DATETIME) UNION ALL
    SELECT CAST('20001226' AS DATETIME) UNION ALL
    SELECT CAST('20001227' AS DATETIME) UNION ALL
    SELECT CAST('20001228' AS DATETIME) UNION ALL
    SELECT CAST('20001229' AS DATETIME) UNION ALL
    SELECT CAST('20001230' AS DATETIME) UNION ALL
    SELECT CAST('20001231' AS DATETIME) UNION ALL
    SELECT CAST('20010101' AS DATETIME) UNION ALL
    SELECT CAST('20010102' AS DATETIME) UNION ALL
    SELECT CAST('20010103' AS DATETIME) UNION ALL
    SELECT CAST('20010104' AS DATETIME) UNION ALL
    SELECT CAST('20010105' AS DATETIME) UNION ALL
    SELECT CAST('20010106' AS DATETIME) UNION ALL
    SELECT CAST('20010107' AS DATETIME) UNION ALL
    SELECT CAST('20010108' AS DATETIME) UNION ALL
    SELECT CAST('20010109' AS DATETIME) UNION ALL
    SELECT CAST('20010110' AS DATETIME) UNION ALL
    SELECT CAST('20010111' AS DATETIME) UNION ALL
    SELECT CAST('20010112' AS DATETIME) UNION ALL
    SELECT CAST('20010113' AS DATETIME) UNION ALL
    SELECT CAST('20010114' AS DATETIME)
), TestDataPlusDOW AS (
    SELECT SomeDate, (@@DATEFIRST + DATEPART(dw, SomeDate) - 1 - 1) % 7 + 1 AS DOW
    FROM TestData
)
SELECT
    FORMAT(SomeDate,                            'ddd yyyy-MM-dd') AS SomeDate,
    FORMAT(DATEADD(dd, -DOW + 1, SomeDate),     'ddd yyyy-MM-dd') AS [Monday],
    FORMAT(DATEADD(dd, -DOW + 1 + 6, SomeDate), 'ddd yyyy-MM-dd') AS [Sunday]
FROM TestDataPlusDOW

Output:

+------------------+------------------+------------------+
|  SomeDate        |  Monday          |    Sunday        |
+------------------+------------------+------------------+
|  Mon 2000-12-25  |  Mon 2000-12-25  |  Sun 2000-12-31  |
|  Tue 2000-12-26  |  Mon 2000-12-25  |  Sun 2000-12-31  |
|  Wed 2000-12-27  |  Mon 2000-12-25  |  Sun 2000-12-31  |
|  Thu 2000-12-28  |  Mon 2000-12-25  |  Sun 2000-12-31  |
|  Fri 2000-12-29  |  Mon 2000-12-25  |  Sun 2000-12-31  |
|  Sat 2000-12-30  |  Mon 2000-12-25  |  Sun 2000-12-31  |
|  Sun 2000-12-31  |  Mon 2000-12-25  |  Sun 2000-12-31  |
|  Mon 2001-01-01  |  Mon 2001-01-01  |  Sun 2001-01-07  |
|  Tue 2001-01-02  |  Mon 2001-01-01  |  Sun 2001-01-07  |
|  Wed 2001-01-03  |  Mon 2001-01-01  |  Sun 2001-01-07  |
|  Thu 2001-01-04  |  Mon 2001-01-01  |  Sun 2001-01-07  |
|  Fri 2001-01-05  |  Mon 2001-01-01  |  Sun 2001-01-07  |
|  Sat 2001-01-06  |  Mon 2001-01-01  |  Sun 2001-01-07  |
|  Sun 2001-01-07  |  Mon 2001-01-01  |  Sun 2001-01-07  |
|  Mon 2001-01-08  |  Mon 2001-01-08  |  Sun 2001-01-14  |
|  Tue 2001-01-09  |  Mon 2001-01-08  |  Sun 2001-01-14  |
|  Wed 2001-01-10  |  Mon 2001-01-08  |  Sun 2001-01-14  |
|  Thu 2001-01-11  |  Mon 2001-01-08  |  Sun 2001-01-14  |
|  Fri 2001-01-12  |  Mon 2001-01-08  |  Sun 2001-01-14  |
|  Sat 2001-01-13  |  Mon 2001-01-08  |  Sun 2001-01-14  |
|  Sun 2001-01-14  |  Mon 2001-01-08  |  Sun 2001-01-14  |
+------------------+------------------+------------------+

* For Sunday to Saturday weeks you need to adjust the equation just a little, like add 1 somewhere.

how to add jquery in laravel project

You can link libraries from cdn (Content delivery network):

<link rel="stylesheet" href="https://maxcdn.bootstrapcdn.com/bootstrap/3.3.5/css/bootstrap.min.css">
<link rel="stylesheet" href="https://maxcdn.bootstrapcdn.com/bootstrap/3.3.5/css/bootstrap-theme.min.css">
<script src="https://ajax.googleapis.com/ajax/libs/jquery/1.11.3/jquery.min.js"></script>
<script src="https://maxcdn.bootstrapcdn.com/bootstrap/3.3.5/js/bootstrap.min.js"></script>

Or link libraries locally, add css files in the css folder and jquery in js folder. You have to keep both folders in the laravel public folder then you can link like below:

<link rel="stylesheet" href="{{asset('css/bootstrap-theme.min.css')}}">
<script src="{{asset('js/jquery.min.js')}}"></script>

or else

{{ HTML::style('css/style.css') }}
{{ HTML::script('js/functions.js') }}

If you link js files and css files locally (like in the last two examples) you need to add js and css files to the js and css folders which are in public\js or public\css not in resources\assets.

Why am I getting a NoClassDefFoundError in Java?

I got this error when I add Maven dependency of another module to my project, the issue was finally solved by add -Xss2m to my program's JVM option(It's one megabyte by default since JDK5.0). It's believed the program does not have enough stack to load class.

What does hash do in python?

You can use the Dictionary data type in python. It's very very similar to the hash—and it also supports nesting, similar to the to nested hash.

Example:

dict = {'Name': 'Zara', 'Age': 7, 'Class': 'First'}
dict['Age'] = 8; # update existing entry
dict['School'] = "DPS School" # Add new entry

print ("dict['Age']: ", dict['Age'])
print ("dict['School']: ", dict['School'])

For more information, please reference this tutorial on the dictionary data type.

Use Robocopy to copy only changed files?

Looks like /e option is what you need, it'll skip same files/directories.

robocopy c:\data c:\backup /e

If you run the command twice, you'll see the second round is much faster since it skips a lot of things.

What is the &#xA; character?

It's a linefeed character. How you use it would be up to you.

Set position / size of UI element as percentage of screen size

The above problem can also be solved using ConstraintLayout through Guidelines.

Below is the snippet.

<?xml version="1.0" encoding="utf-8"?>
<android.support.constraint.ConstraintLayout xmlns:android="http://schemas.android.com/apk/res/android"
xmlns:app="http://schemas.android.com/apk/res-auto"
xmlns:tools="http://schemas.android.com/tools"
android:layout_width="match_parent"
android:layout_height="match_parent">

<android.support.constraint.Guideline
    android:id="@+id/upperGuideLine"
    android:layout_width="wrap_content"
    android:layout_height="wrap_content"
    android:orientation="horizontal"
    app:layout_constraintGuide_percent="0.68" />

<Gallery
    android:id="@+id/gallery"
    android:layout_width="0dp"
    android:layout_height="0dp"
    app:layout_constraintBottom_toTopOf="@+id/lowerGuideLine"
    app:layout_constraintEnd_toEndOf="parent"
    app:layout_constraintStart_toStartOf="parent"
    app:layout_constraintTop_toTopOf="@+id/upperGuideLine" />

<android.support.constraint.Guideline
    android:id="@+id/lowerGuideLine"
    android:layout_width="wrap_content"
    android:layout_height="wrap_content"
    android:orientation="horizontal"
    app:layout_constraintGuide_percent="0.84" />

</android.support.constraint.ConstraintLayout>

Android XML Percent Symbol

To allow the app using formatted strings from resources you should correct your xml. So, for example

<string name="app_name">Your App name, ver.%d</string>

should be replaced with

<string name="app_name">Your App name, ver.%1$d</string>

You can see this for details.

WPF Check box: Check changed handling

I know this is an old question, but how about just binding to Command if using MVVM?

ex:

<CheckBox Content="Case Sensitive" Command="{Binding bSearchCaseSensitive}"/>

For me it triggers on both Check and Uncheck.

Resolve absolute path from relative path and/or file name

In batch files, as in standard C programs, argument 0 contains the path to the currently executing script. You can use %~dp0 to get only the path portion of the 0th argument (which is the current script) - this path is always a fully qualified path.

You can also get the fully qualified path of your first argument by using %~f1, but this gives a path according to the current working directory, which is obviously not what you want.

Personally, I often use the %~dp0%~1 idiom in my batch file, which interpret the first argument relative to the path of the executing batch. It does have a shortcoming though: it miserably fails if the first argument is fully-qualified.

If you need to support both relative and absolute paths, you can make use of Frédéric Ménez's solution: temporarily change the current working directory.

Here's an example that'll demonstrate each of these techniques:

@echo off
echo %%~dp0 is "%~dp0"
echo %%0 is "%0"
echo %%~dpnx0 is "%~dpnx0"
echo %%~f1 is "%~f1"
echo %%~dp0%%~1 is "%~dp0%~1"

rem Temporarily change the current working directory, to retrieve a full path 
rem   to the first parameter
pushd .
cd %~dp0
echo batch-relative %%~f1 is "%~f1"
popd

If you save this as c:\temp\example.bat and the run it from c:\Users\Public as

c:\Users\Public>\temp\example.bat ..\windows

...you'll observe the following output:

%~dp0 is "C:\temp\"
%0 is "\temp\example.bat"
%~dpnx0 is "C:\temp\example.bat"
%~f1 is "C:\Users\windows"
%~dp0%~1 is "C:\temp\..\windows"
batch-relative %~f1 is "C:\Windows"

the documentation for the set of modifiers allowed on a batch argument can be found here: https://docs.microsoft.com/en-us/windows-server/administration/windows-commands/call

Android Camera Preview Stretched

i tried all the solution above but none of them works for me. finaly i solved it myself, and find actually it's quite easy. there are two points you need to be careful.

parameters.setPreviewSize(cameraResolution.x, cameraResolution.y);

this previewSize must be one of the camera supported resolution, which can be get as below:

List<Camera.Size> rawSupportedSizes = parameters.getSupportedPreviewSizes(); 

usually one of the rawSupportedSize equals to the device resolution.

Second, place your SurfaceView in a FrameLayout and set the surface layout height and width in surfaceChanged method as above

FrameLayout.LayoutParams layoutParams = (FrameLayout.LayoutParams) surfaceView.getLayoutParams();
layoutParams.height = cameraResolution.x;
layoutParams.width = cameraResolution.y;

Ok, things done, hope this could help you.

How to generate sample XML documents from their DTD or XSD?

The OpenXSD library mentions that they have support for generating XML instances based on the XSD. Check that out.

Response Buffer Limit Exceeded

If you are looking for the reason and don't want to fight the system settings, these are two major situations I faced:

  1. You may have an infinite loop without next or recordest.movenext
  2. Your text data is very large but you think it is not! The common reason for this situation is to copy-paste an Image from Microsoft word directly into the editor and so the server translates the image to data objects and saves it in your text field. This can easily occupies the database resources and causes buffer problem when you call the data again.

tsc is not recognized as internal or external command

You need to run:

npx tsc

...rather than just calling tsc own its on like a Windows command as everyone else seems to be suggesting.

If you don't have npx installed then you should. It should be installed globally (unlike Typescript). So first run:

npm install -g npx

..then run npx tsc.

Get docker container id from container name

  1. Get container Ids of running containers ::

    $docker ps -qf "name=IMAGE_NAME"
    
        -f: Filter output based on conditions provided
        -q: Only display numeric container IDs
    
  2. Get container Ids of all containers ::

    $docker ps -aqf "name=IMAGE_NAME"
    
        -a: all containers
    

MySQL my.ini location

For MySql Server 8.0 The default location is %WINDIR% or C:\Windows.

You need to add a "my.ini" file there.

Here's a sample of what I put in the ini file.

[mysqld]

secure_file_priv=""

Make sure to restart the MySQL service after that.

Can't find keyplane that supports type 4 for keyboard iPhone-Portrait-NumberPad; using 3876877096_Portrait_iPhone-Simple-Pad_Default

I had the same problem using distribution provisioning profile. Check that you use developer profile

How can I view all historical changes to a file in SVN

Slightly different from what you described, but I think this might be what you actually need:

svn blame filename

It will print the file with each line prefixed by the time and author of the commit that last changed it.

CSS class for pointer cursor

As of June 2020, adding role='button' to any HTML tag would add cursor: "pointer" to the element styling.

<span role="button">Non-button element button</span>

Official discussion on this feature - https://github.com/twbs/bootstrap/issues/23709

Documentation link - https://getbootstrap.com/docs/4.5/content/reboot/#pointers-on-buttons

How can I find all of the distinct file extensions in a folder hierarchy?

I've found it simple and fast...

   # find . -type f -exec basename {} \; | awk -F"." '{print $NF}' > /tmp/outfile.txt
   # cat /tmp/outfile.txt | sort | uniq -c| sort -n > tmp/outfile_sorted.txt

XML Schema How to Restrict Attribute by Enumeration

The numerical value seems to be missing from your price definition. Try the following:

<xs:simpleType name="curr">
  <xs:restriction base="xs:string">
    <xs:enumeration value="pounds" />
    <xs:enumeration value="euros" />
    <xs:enumeration value="dollars" />
  </xs:restriction>
</xs:simpleType>



<xs:element name="price">
        <xs:complexType>
            <xs:extension base="xs:decimal">
              <xs:attribute name="currency" type="curr"/>
            </xs:extension>
        </xs:complexType>
</xs:element>

How do I filter ForeignKey choices in a Django ModelForm?

ForeignKey is represented by django.forms.ModelChoiceField, which is a ChoiceField whose choices are a model QuerySet. See the reference for ModelChoiceField.

So, provide a QuerySet to the field's queryset attribute. Depends on how your form is built. If you build an explicit form, you'll have fields named directly.

form.rate.queryset = Rate.objects.filter(company_id=the_company.id)

If you take the default ModelForm object, form.fields["rate"].queryset = ...

This is done explicitly in the view. No hacking around.

Creating a class object in c++

First of all, both cases calls a constructor. If you write

Example *example = new Example();

then you are creating an object, call the constructor and retrieve a pointer to it.

If you write

Example example;

The only difference is that you are getting the object and not a pointer to it. The constructor called in this case is the same as above, the default (no argument) constructor.

As for the singleton question, you must simple invoke your static method by writing:

Example *e = Singleton::getExample();

Setting active profile and config location from command line in spring boot

you can use the following command line:

java -jar -Dspring.profiles.active=[yourProfileName] target/[yourJar].jar

Possible to iterate backwards through a foreach?

No. ForEach just iterates through collection for each item and order depends whether it uses IEnumerable or GetEnumerator().

How to set the maximum memory usage for JVM?

The NativeHeap can be increasded by -XX:MaxDirectMemorySize=256M (default is 128)

I've never used it. Maybe you'll find it useful.

How to convert int to float in C?

This can give you the correct Answer

#include <stdio.h>
int main()
{
    float total=100, number=50;
    float percentage;
    percentage=(number/total)*100;
    printf("%0.2f",percentage);
    return 0;
}

How to use classes from .jar files?

Not every jar file is executable.

Now, you need to import the classes, which are there under the jar, in your java file. For example,

import org.xml.sax.SAXException;

If you are working on an IDE, then you should refer its documentation. Or at least specify which one you are using here in this thread. It would definitely enable us to help you further.

And if you are not using any IDE, then please look at javac -cp option. However, it's much better idea to package your program in a jar file, and include all the required jars within that. Then, in order to execute your jar, like,

java -jar my_program.jar

you should have a META-INF/MANIFEST.MF file in your jar. See here, for how-to.

WSDL vs REST Pros and Cons

The toolset on the client side would be one. And the familiarity with SOAP services the other. More and more services are going the RESTful route these days, and testing such services can be done with simple cURL examples. Although, it's not all that difficult to implement both methods and allow for the widest utilization from clients.

If you need to pick one, I'd suggest REST, it's easier.

What is aria-label and how should I use it?

In the example you give, you're perfectly right, you have to set the title attribute.

If the aria-label is one tool used by assistive technologies (like screen readers), it is not natively supported on browsers and has no effect on them. It won't be of any help to most of the people targetted by the WCAG (except screen reader users), for instance a person with intellectal disabilities.

The "X" is not sufficient enough to give information to the action led by the button (think about someone with no computer knowledge). It might mean "close", "delete", "cancel", "reduce", a strange cross, a doodle, nothing.

Despite the fact that the W3C seems to promote the aria-label rather that the title attribute here: http://www.w3.org/TR/2014/NOTE-WCAG20-TECHS-20140916/ARIA14 in a similar example, you can see that the technology support does not include standard browsers : http://www.w3.org/WAI/WCAG20/Techniques/ua-notes/aria#ARIA14

In fact aria-label, in this exact situation might be used to give more context to an action:

For instance, blind people do not perceive popups like those of us with good vision, it's like a change of context. "Back to the page" will be a more convenient alternative for a screen reader, when "Close" is more significant for someone with no screen reader.

  <button
      aria-label="Back to the page"
      title="Close" onclick="myDialog.close()">X</button>

not None test in Python

From, Programming Recommendations, PEP 8:

Comparisons to singletons like None should always be done with is or is not, never the equality operators.

Also, beware of writing if x when you really mean if x is not None — e.g. when testing whether a variable or argument that defaults to None was set to some other value. The other value might have a type (such as a container) that could be false in a boolean context!

PEP 8 is essential reading for any Python programmer.

Difference between e.target and e.currentTarget

e.currentTarget is always the element the event is actually bound do. e.target is the element the event originated from, so e.target could be a child of e.currentTarget, or e.target could be === e.currentTarget, depending on how your markup is structured.

Javascript .querySelector find <div> by innerTEXT

Use XPath and document.evaluate(), and make sure to use text() and not . for the contains() argument, or else you will have the entire HTML, or outermost div element matched.

var headings = document.evaluate("//h1[contains(text(), 'Hello')]", document, null, XPathResult.ANY_TYPE, null );

or ignore leading and trailing whitespace

var headings = document.evaluate("//h1[contains(normalize-space(text()), 'Hello')]", document, null, XPathResult.ANY_TYPE, null );

or match all tag types (div, h1, p, etc.)

var headings = document.evaluate("//*[contains(text(), 'Hello')]", document, null, XPathResult.ANY_TYPE, null );

Then iterate

let thisHeading;
while(thisHeading = headings.iterateNext()){
    // thisHeading contains matched node
}

Passing data from controller to view in Laravel

try with this code :

Controller:
-----------------------------
 $fromdate=date('Y-m-d',strtotime(Input::get('fromdate'))); 
        $todate=date('Y-m-d',strtotime(Input::get('todate'))); 

 $datas=array('fromdate'=>"From Date :".date('d-m-Y',strtotime($fromdate)), 'todate'=>"To 
        return view('inventoryreport/inventoryreportview', compact('datas'));

View Page : 
@foreach($datas as $student)
   {{$student}}

@endforeach
[Link here]

Installing MySQL Python on Mac OS X

As others mentioned before me....getting Python to work with MySQL on a Mac is a ?@#$@&%^!! nightmare.

Installed Django framework on Mac OS 10.7.5 initially from the original Django website and when the MySQLdb didn't work, and after many hours googling and trying solutions from SO, I have installed the Django stack from BitNami http://bitnami.com/stack/django

Still, got the issues mentioned above and then some more...

What helped me eventually is what Josh recommends on his blog: http://joshbranchaud.com/blog/2013/02/10/Errors-While-Setting-Up-Django.html

Now Python 2.7 is finally connected to MySQL 5.5

Log4j2 configuration - No log4j2 configuration file found

In my case I had to put it in the bin folder of my project even the fact that my classpath is set to the src folder. I have no idea why, but it's worth a try.

How to access at request attributes in JSP?

Just noting this here in case anyone else has a similar issue.
If you're directing a request directly to a JSP, using Apache Tomcat web.xml configuration, then ${requestScope.attr} doesn't seem to work, instead ${param.attr} contains the request attribute attr.

How to center content in a bootstrap column?

col-lg-4 col-md-6 col-sm-8 col-11 mx-auto

 1. col-lg-4 = 1200px (popular 1366, 1600, 1920+)
 2. col-md-6 = 970px (popular 1024, 1200)
 3. col-sm-8 = 768px (popular 800, 768)
 4. col-11 set default smaller devices for gutter (popular 600,480,414,375,360,312)
 5. mx-auto = always block center


react-router getting this.props.location in child components

(Update) V5.1 & Hooks (Requires React >= 16.8)

You can use useHistory, useLocation and useRouteMatch in your component to get match, history and location .

const Child = () => {
  const location = useLocation();
  const history = useHistory();
  const match = useRouteMatch("write-the-url-you-want-to-match-here");

  return (
    <div>{location.pathname}</div>
  )
}

export default Child

(Update) V4 & V5

You can use withRouter HOC in order to inject match, history and location in your component props.

class Child extends React.Component {
  static propTypes = {
    match: PropTypes.object.isRequired,
    location: PropTypes.object.isRequired,
    history: PropTypes.object.isRequired
  }

  render() {
    const { match, location, history } = this.props

    return (
      <div>{location.pathname}</div>
    )
  }
}

export default withRouter(Child)

(Update) V3

You can use withRouter HOC in order to inject router, params, location, routes in your component props.

class Child extends React.Component {

  render() {
    const { router, params, location, routes } = this.props

    return (
      <div>{location.pathname}</div>
    )
  }
}

export default withRouter(Child)

Original answer

If you don't want to use the props, you can use the context as described in React Router documentation

First, you have to set up your childContextTypes and getChildContext

class App extends React.Component{

  getChildContext() {
    return {
      location: this.props.location
    }
  }

  render() {
    return <Child/>;
  }
}

App.childContextTypes = {
    location: React.PropTypes.object
}

Then, you will be able to access to the location object in your child components using the context like this

class Child extends React.Component{

   render() {
     return (
       <div>{this.context.location.pathname}</div>
     )
   }

}

Child.contextTypes = {
    location: React.PropTypes.object
 }

long long int vs. long int vs. int64_t in C++

You don't need to go to 64-bit to see something like this. Consider int32_t on common 32-bit platforms. It might be typedef'ed as int or as a long, but obviously only one of the two at a time. int and long are of course distinct types.

It's not hard to see that there is no workaround which makes int == int32_t == long on 32-bit systems. For the same reason, there's no way to make long == int64_t == long long on 64-bit systems.

If you could, the possible consequences would be rather painful for code that overloaded foo(int), foo(long) and foo(long long) - suddenly they'd have two definitions for the same overload?!

The correct solution is that your template code usually should not be relying on a precise type, but on the properties of that type. The whole same_type logic could still be OK for specific cases:

long foo(long x);
std::tr1::disable_if(same_type(int64_t, long), int64_t)::type foo(int64_t);

I.e., the overload foo(int64_t) is not defined when it's exactly the same as foo(long).

[edit] With C++11, we now have a standard way to write this:

long foo(long x);
std::enable_if<!std::is_same<int64_t, long>::value, int64_t>::type foo(int64_t);

[edit] Or C++20

long foo(long x);
int64_t foo(int64_t) requires (!std::is_same_v<int64_t, long>);

What jar should I include to use javax.persistence package in a hibernate based application?

hibernate.jar and hibernate-entitymanager.jar contains only the packages org.hibernate.*. So you should take it from the Glassfish project.

how to save DOMPDF generated content to file?

I have just used dompdf and the code was a little different but it worked.

Here it is:

require_once("./pdf/dompdf_config.inc.php");
$files = glob("./pdf/include/*.php");
foreach($files as $file) include_once($file);

$html =
      '<html><body>'.
      '<p>Put your html here, or generate it with your favourite '.
      'templating system.</p>'.
      '</body></html>';

    $dompdf = new DOMPDF();
    $dompdf->load_html($html);
    $dompdf->render();
    $output = $dompdf->output();
    file_put_contents('Brochure.pdf', $output);

Only difference here is that all of the files in the include directory are included.

Other than that my only suggestion would be to specify a full directory path for writing the file rather than just the filename.

Animate scroll to ID on page load

for simple Scroll, use following style

height: 200px; overflow: scroll;

and use this style class which div or section you want to show scroll

How to hide a div after some time period?

In older versions of jquery you'll have to do it the "javascript way" using settimeout

setTimeout( function(){$('div').hide();} , 4000);

or

setTimeout( "$('div').hide();", 4000);

Recently with jquery 1.4 this solution has been added:

$("div").delay(4000).hide();

Of course replace "div" by the correct element using a valid jquery selector and call the function when the document is ready.

How to get Latitude and Longitude of the mobile device in android?

Use the LocationManager.

LocationManager lm = (LocationManager)getSystemService(Context.LOCATION_SERVICE); 
Location location = lm.getLastKnownLocation(LocationManager.GPS_PROVIDER);
double longitude = location.getLongitude();
double latitude = location.getLatitude();

The call to getLastKnownLocation() doesn't block - which means it will return null if no position is currently available - so you probably want to have a look at passing a LocationListener to the requestLocationUpdates() method instead, which will give you asynchronous updates of your location.

private final LocationListener locationListener = new LocationListener() {
    public void onLocationChanged(Location location) {
        longitude = location.getLongitude();
        latitude = location.getLatitude();
    }
}

lm.requestLocationUpdates(LocationManager.GPS_PROVIDER, 2000, 10, locationListener);

You'll need to give your application the ACCESS_FINE_LOCATION permission if you want to use GPS.

<uses-permission android:name="android.permission.ACCESS_FINE_LOCATION" />

You may also want to add the ACCESS_COARSE_LOCATION permission for when GPS isn't available and select your location provider with the getBestProvider() method.

How do I create a Bash alias?

On OS X you want to use ~/.bash_profile. This is because by default Terminal.app opens a login shell for each new window.

See more about the different configuration files and when they are used here: What's the difference between .bashrc, .bash_profile, and .environment?

and in relation to OSX here: About .bash_profile, .bashrc, and where should alias be written in?

What is std::move(), and when should it be used?

"What is it?" and "What does it do?" has been explained above.

I will give a example of "when it should be used".

For example, we have a class with lots of resource like big array in it.

class ResHeavy{ //  ResHeavy means heavy resource
    public:
        ResHeavy(int len=10):_upInt(new int[len]),_len(len){
            cout<<"default ctor"<<endl;
        }

        ResHeavy(const ResHeavy& rhs):_upInt(new int[rhs._len]),_len(rhs._len){
            cout<<"copy ctor"<<endl;
        }

        ResHeavy& operator=(const ResHeavy& rhs){
            _upInt.reset(new int[rhs._len]);
            _len = rhs._len;
            cout<<"operator= ctor"<<endl;
        }

        ResHeavy(ResHeavy&& rhs){
            _upInt = std::move(rhs._upInt);
            _len = rhs._len;
            rhs._len = 0;
            cout<<"move ctor"<<endl;
        }

    // check array valid
    bool is_up_valid(){
        return _upInt != nullptr;
    }

    private:
        std::unique_ptr<int[]> _upInt; // heavy array resource
        int _len; // length of int array
};

Test code:

void test_std_move2(){
    ResHeavy rh; // only one int[]
    // operator rh

    // after some operator of rh, it becomes no-use
    // transform it to other object
    ResHeavy rh2 = std::move(rh); // rh becomes invalid

    // show rh, rh2 it valid
    if(rh.is_up_valid())
        cout<<"rh valid"<<endl;
    else
        cout<<"rh invalid"<<endl;

    if(rh2.is_up_valid())
        cout<<"rh2 valid"<<endl;
    else
        cout<<"rh2 invalid"<<endl;

    // new ResHeavy object, created by copy ctor
    ResHeavy rh3(rh2);  // two copy of int[]

    if(rh3.is_up_valid())
        cout<<"rh3 valid"<<endl;
    else
        cout<<"rh3 invalid"<<endl;
}

output as below:

default ctor
move ctor
rh invalid
rh2 valid
copy ctor
rh3 valid

We can see that std::move with move constructor makes transform resource easily.

Where else is std::move useful?

std::move can also be useful when sorting an array of elements. Many sorting algorithms (such as selection sort and bubble sort) work by swapping pairs of elements. Previously, we’ve had to resort to copy-semantics to do the swapping. Now we can use move semantics, which is more efficient.

It can also be useful if we want to move the contents managed by one smart pointer to another.

Cited:

what does the __file__ variable mean/do?

When a module is loaded from a file in Python, __file__ is set to its path. You can then use that with other functions to find the directory that the file is located in.

Taking your examples one at a time:

A = os.path.join(os.path.dirname(__file__), '..')
# A is the parent directory of the directory where program resides.

B = os.path.dirname(os.path.realpath(__file__))
# B is the canonicalised (?) directory where the program resides.

C = os.path.abspath(os.path.dirname(__file__))
# C is the absolute path of the directory where the program resides.

You can see the various values returned from these here:

import os
print(__file__)
print(os.path.join(os.path.dirname(__file__), '..'))
print(os.path.dirname(os.path.realpath(__file__)))
print(os.path.abspath(os.path.dirname(__file__)))

and make sure you run it from different locations (such as ./text.py, ~/python/text.py and so forth) to see what difference that makes.


I just want to address some confusion first. __file__ is not a wildcard it is an attribute. Double underscore attributes and methods are considered to be "special" by convention and serve a special purpose.

http://docs.python.org/reference/datamodel.html shows many of the special methods and attributes, if not all of them.

In this case __file__ is an attribute of a module (a module object). In Python a .py file is a module. So import amodule will have an attribute of __file__ which means different things under difference circumstances.

Taken from the docs:

__file__ is the pathname of the file from which the module was loaded, if it was loaded from a file. The __file__ attribute is not present for C modules that are statically linked into the interpreter; for extension modules loaded dynamically from a shared library, it is the pathname of the shared library file.

In your case the module is accessing it's own __file__ attribute in the global namespace.

To see this in action try:

# file: test.py

print globals()
print __file__

And run:

python test.py

{'__builtins__': <module '__builtin__' (built-in)>, '__name__': '__main__', '__file__':
 'test_print__file__.py', '__doc__': None, '__package__': None}
test_print__file__.py

Jquery: how to trigger click event on pressing enter key

This appear to be default behaviour now, so it's enough to do:

$("#press-enter").on("click", function(){alert("You `clicked' or 'Entered' me!")})

You can try it in this JSFiddle

Tested on: Chrome 56.0 and Firefox (Dev Edition) 54.0a2, both with jQuery 2.2.x and 3.x

Javascript string/integer comparisons

The alert() wants to display a string, so it will interpret "2">"10" as a string.

Use the following:

var greater = parseInt("2") > parseInt("10");
alert("Is greater than? " + greater);

var less = parseInt("2") < parseInt("10");
alert("Is less than? " + less);

How to put img inline with text

This should display the image inline:

.content-dir-item img.mail {
    display: inline-block;
    *display: inline; /* for older IE */
    *zoom: 1; /* for older IE */
}

What is the equivalent of Java static methods in Kotlin?

This also worked for me

object Bell {
    @JvmStatic
    fun ring() { }
}

from Kotlin

Bell.ring()

from Java

Bell.ring()

difference between css height : 100% vs height : auto

The default is height: auto in browser, but height: X% Defines the height in percentage of the containing block.

Excel formula to reference 'CELL TO THE LEFT'

When creating your conditional formatting, set the range to which it applies to what you want (the whole sheet), then enter a relative formula (remove the $ signs) as if you were only formatting the upper-left corner.

Excel will properly apply the formatting to the rest of the cells accordingly.

In this example, starting in B1, the left cell would be A1. Just use that--no advanced formula required.


If you're looking for something more advanced, you can play around with column(), row(), and indirect(...).

How do I add a placeholder on a CharField in Django?

Most of the time I just wish to have all placeholders equal to the verbose name of the field defined in my models

I've added a mixin to easily do this to any form that I create,

class ProductForm(PlaceholderMixin, ModelForm):
    class Meta:
        model = Product
        fields = ('name', 'description', 'location', 'store')

And

class PlaceholderMixin:
    def __init__(self, *args, **kwargs):
        super().__init__(*args, **kwargs)
        field_names = [field_name for field_name, _ in self.fields.items()]
        for field_name in field_names:
            field = self.fields.get(field_name)
            field.widget.attrs.update({'placeholder': field.label})

Passing parameter to controller from route in laravel

You can add them like this

  Route::get('company/{name}', 'PublicareaController@companydetails');

Printing prime numbers from 1 through 100

I did it in perl based on the most popular answer by ProdigySim's 2nd method. I had to add a break equivalent in perl, last, right after the print $i . " \n"; to avoid outputting the primes twice.

#!/bin/perl
use strict;

for(my $i=2; $i < 100; $i++){

    my $prime = 1;

    for (my $j=2; $j*$j<=$i; $j++){
        if ($i % $j == 0){
            $prime = 0;
            last;
        }
        if($prime) {
            print $i . " \n";
            last;
        }
    }

}

Using .htaccess to make all .html pages to run as .php files?

In My Godaddy Server the following code worked

Options +ExecCGI
AddType application/x-httpd-php .php .html
AddHandler x-httpd-php5 .php .html

Get the filename of a fileupload in a document through JavaScript

Using code like this in a form I can capture the original source upload filename, copy it to a second simple input field. This is so user can provide an alternate upload filename in submit request since the file upload filename is immutable.

    <input type="file" id="imgup1" name="imagefile">
      onchange="document.getElementsByName('imgfn1')[0].value = document.getElementById('imgup1').value;">
    <input type="text" name="imgfn1" value="">

Setting up Eclipse with JRE Path

Java version used : 1.8 IDE : Eclipse Neon

Adding like the below didn't work for me

-vm [relative java home]/jdk1.8.0_21/bin/javaw.exe

and then when i removed

javaw.exe

it worked, so it will be like

-vm [relative java home]/jdk1.8.0_21/bin/

How can I get the current user's username in Bash?

In Solaris OS I used this command:

$ who am i     # Remember to use it with space.

On Linux- Someone already answered this in comments.

$ whoami       # Without space

How to iterate through SparseArray?

For whoever is using Kotlin, honestly the by far easiest way to iterate over a SparseArray is: Use the Kotlin extension from Anko or Android KTX! (credit to Yazazzello for pointing out Android KTX)

Simply call forEach { i, item -> }

How to use an existing database with an Android application

NOTE: Before trying this code, please find this line in the below code:

private static String DB_NAME ="YourDbName"; // Database name

DB_NAME here is the name of your database. It is assumed that you have a copy of the database in the assets folder, so for example, if your database name is ordersDB, then the value of DB_NAME will be ordersDB,

private static String DB_NAME ="ordersDB";

Keep the database in assets folder and then follow the below:

DataHelper class:

import java.io.File;
import java.io.FileOutputStream;
import java.io.IOException;
import java.io.InputStream;
import java.io.OutputStream;

import android.content.Context;
import android.database.SQLException;
import android.database.sqlite.SQLiteDatabase;
import android.database.sqlite.SQLiteOpenHelper;
import android.util.Log;

public class DataBaseHelper extends SQLiteOpenHelper {

    private static String TAG = "DataBaseHelper"; // Tag just for the LogCat window
    private static String DB_NAME ="YourDbName"; // Database name
    private static int DB_VERSION = 1; // Database version
    private final File DB_FILE;
    private SQLiteDatabase mDataBase;
    private final Context mContext;

    public DataBaseHelper(Context context) {
        super(context, DB_NAME, null, DB_VERSION);
        DB_FILE = context.getDatabasePath(DB_NAME);
        this.mContext = context;
    }

    public void createDataBase() throws IOException {
        // If the database does not exist, copy it from the assets.
        boolean mDataBaseExist = checkDataBase();
        if(!mDataBaseExist) {
            this.getReadableDatabase();
            this.close();
            try {
                // Copy the database from assests
                copyDataBase();
                Log.e(TAG, "createDatabase database created");
            } catch (IOException mIOException) {
                throw new Error("ErrorCopyingDataBase");
            }
        }
    }

    // Check that the database file exists in databases folder
    private boolean checkDataBase() {
        return DB_FILE.exists();
    }

    // Copy the database from assets
    private void copyDataBase() throws IOException {
        InputStream mInput = mContext.getAssets().open(DB_NAME);
        OutputStream mOutput = new FileOutputStream(DB_FILE);
        byte[] mBuffer = new byte[1024];
        int mLength;
        while ((mLength = mInput.read(mBuffer)) > 0) {
            mOutput.write(mBuffer, 0, mLength);
        }
        mOutput.flush();
        mOutput.close();
        mInput.close();
    }

    // Open the database, so we can query it
    public boolean openDataBase() throws SQLException {
        // Log.v("DB_PATH", DB_FILE.getAbsolutePath());
        mDataBase = SQLiteDatabase.openDatabase(DB_FILE, null, SQLiteDatabase.CREATE_IF_NECESSARY);
        // mDataBase = SQLiteDatabase.openDatabase(DB_FILE, null, SQLiteDatabase.NO_LOCALIZED_COLLATORS);
        return mDataBase != null;
    }

    @Override
    public synchronized void close() {
        if(mDataBase != null) {
            mDataBase.close();
        }
        super.close();
    }

}

Write a DataAdapter class like:

import java.io.IOException;
import android.content.Context;
import android.database.Cursor;
import android.database.SQLException;
import android.database.sqlite.SQLiteDatabase;
import android.util.Log;

public class TestAdapter {

    protected static final String TAG = "DataAdapter";

    private final Context mContext;
    private SQLiteDatabase mDb;
    private DataBaseHelper mDbHelper;

    public TestAdapter(Context context) {
        this.mContext = context;
        mDbHelper = new DataBaseHelper(mContext);
    }

    public TestAdapter createDatabase() throws SQLException {
        try {
            mDbHelper.createDataBase();
        } catch (IOException mIOException) {
            Log.e(TAG, mIOException.toString() + "  UnableToCreateDatabase");
            throw new Error("UnableToCreateDatabase");
        }
        return this;
    }

    public TestAdapter open() throws SQLException {
        try {
            mDbHelper.openDataBase();
            mDbHelper.close();
            mDb = mDbHelper.getReadableDatabase();
        } catch (SQLException mSQLException) {
            Log.e(TAG, "open >>"+ mSQLException.toString());
            throw mSQLException;
        }
        return this;
    }

    public void close() {
        mDbHelper.close();
    }

     public Cursor getTestData() {
         try {
             String sql ="SELECT * FROM myTable";
             Cursor mCur = mDb.rawQuery(sql, null);
             if (mCur != null) {
                mCur.moveToNext();
             }
             return mCur;
         } catch (SQLException mSQLException) {
             Log.e(TAG, "getTestData >>"+ mSQLException.toString());
             throw mSQLException;
         }
     }
}

Now you can use it like:

TestAdapter mDbHelper = new TestAdapter(urContext);
mDbHelper.createDatabase();
mDbHelper.open();

Cursor testdata = mDbHelper.getTestData();

mDbHelper.close();

EDIT: Thanks to JDx

For Android 4.1 (Jelly Bean), change:

DB_PATH = "/data/data/" + context.getPackageName() + "/databases/";

to:

DB_PATH = context.getApplicationInfo().dataDir + "/databases/";

in the DataHelper class, this code will work on Jelly Bean 4.2 multi-users.

EDIT: Instead of using hardcoded path, we can use

DB_PATH = context.getDatabasePath(DB_NAME).getAbsolutePath();

which will give us the full path to the database file and works on all Android versions

Specifying Style and Weight for Google Fonts

They use regular CSS.

Just use your regular font family like this:

font-family: 'Open Sans', sans-serif;

Now you decide what "weight" the font should have by adding

for semi-bold

font-weight:600;

for bold (700)

font-weight:bold;

for extra bold (800)

font-weight:800;

Like this its fallback proof, so if the google font should "fail" your backup font Arial/Helvetica(Sans-serif) use the same weight as the google font.

Pretty smart :-)

Note that the different font weights have to be specifically imported via the link tag url (family query param of the google font url) in the header.

For example the following link will include both weights 400 and 700:

<link href='fonts.googleapis.com/css?family=Comfortaa:400,700'; rel='stylesheet' type='text/css'>

How to have Android Service communicate with Activity

As mentioned by Madhur, you can use a bus for communication.

In case of using a Bus you have some options:

Otto event Bus library (deprecated in favor of RxJava)

http://square.github.io/otto/

Green Robot’s EventBus

http://greenrobot.org/eventbus/

NYBus (RxBus, implemented using RxJava. very similar to the EventBus)

https://github.com/MindorksOpenSource/NYBus

Parsing GET request parameters in a URL that contains another URL

if (isset($_SERVER['HTTPS'])){
    echo "https://$_SERVER[HTTP_HOST]$_SERVER[REQUEST_URI]$_SERVER[QUERY_STRING]";
}else{
    echo "http://$_SERVER[HTTP_HOST]$_SERVER[REQUEST_URI]$_SERVER[QUERY_STRING]";
}

Auto margins don't center image in page

Under some circumstances (such as earlier versions of IE, Gecko, Webkit) and inheritance, elements with position:relative; will prevent margin:0 auto; from working, even if top, right, bottom, and left aren't set.

Setting the element to position:static; (the default) may fix it under these circumstances. Generally, block level elements with a specified width will respect margin:0 auto; using either relative or static positioning.

How to Execute stored procedure from SQL Plus?

You forgot to put z as an bind variable.

The following EXECUTE command runs a PL/SQL statement that references a stored procedure:

SQL> EXECUTE -
> :Z := EMP_SALE.HIRE('JACK','MANAGER','JONES',2990,'SALES')

Note that the value returned by the stored procedure is being return into :Z

How to build jars from IntelliJ properly?

Here is the official answer of IntelliJ IDEA 2018.3 Help. I tried and It worked.

To build a JAR file from a module;

  1. On the main menu, choose Build | Build Artifact.

  2. From the drop-down list, select the desired artifact of the type JAR. The list shows all the artifacts configured for the current project. To have all the configured artifacts built, choose the Build all artifacts option.

Twitter bootstrap scrollable modal

Your modal is being hidden in firefox, and that is because of the negative margin declaration you have inside your general stylesheet:

.modal {
    margin-top: -45%; /* remove this */
    max-height: 90%;
    overflow-y: auto;
}

Remove the negative margin and everything works just fine.

Invalid postback or callback argument. Event validation is enabled using '<pages enableEventValidation="true"/>'

If you fill the DropdownList through client side script then clear the list before submit the form back to server; then ASP.NET will not complain and the security will be still on.

And to get the data selected from the DDL, you can attach an "OnChange" event to the DDL to collect the value in a hidden Input or in a textbox with Style="display: none;"

filter: progid:DXImageTransform.Microsoft.gradient is not working in ie7

This should work:

background: -moz-linear-gradient(center top , #fad59f, #fa9907) repeat scroll 0 0 transparent;
 /* For WebKit (Safari, Google Chrome etc) */
background: -webkit-gradient(linear, left top, left bottom, from(#fad59f), to(#fa9907));
/* For Mozilla/Gecko (Firefox etc) */
background: -moz-linear-gradient(top, #fad59f, #fa9907);
/* For Internet Explorer 5.5 - 7 */
filter: progid:DXImageTransform.Microsoft.gradient(startColorstr=#fad59f, endColorstr=#fa9907);
/* For Internet Explorer 8 */
-ms-filter: "progid:DXImageTransform.Microsoft.gradient(startColorstr=#fad59f, endColorstr=#fa9907)";

Otherwise generate using the following link and get the code.

http://www.colorzilla.com/gradient-editor/

Raw_Input() Is Not Defined

For Python 3.x, use input(). For Python 2.x, use raw_input(). Don't forget you can add a prompt string in your input() call to create one less print statement. input("GUESS THAT NUMBER!").

AngularJs .$setPristine to reset form

DavidLn's answer has worked well for me in the past. But it doesn't capture all of setPristine's functionality, which tripped me up this time. Here is a fuller shim:

var form_set_pristine = function(form){
    // 2013-12-20 DF TODO: remove this function on Angular 1.1.x+ upgrade
    // function is included natively

    if(form.$setPristine){
        form.$setPristine();
    } else {
        form.$pristine = true;
        form.$dirty = false;
        angular.forEach(form, function (input, key) {
            if (input.$pristine)
                input.$pristine = true;
            if (input.$dirty) {
                input.$dirty = false;
            }
        });
    }
};

Streaming video from Android camera to server

I've built an open-source SDK called Kickflip to make streaming video from Android a painless experience.

The SDK demonstrates use of Android 4.3's MediaCodec API to direct the device hardware encoder's packets directly to FFmpeg for RTMP (with librtmp) or HLS streaming of H.264 / AAC. It also demonstrates realtime OpenGL Effects (titling, chroma key, fades) and background recording.

Thanks SO, and especially, fadden.

Best practice for instantiating a new Android Fragment

I disagree with yydi answer saying:

If Android decides to recreate your Fragment later, it's going to call the no-argument constructor of your fragment. So overloading the constructor is not a solution.

I think it is a solution and a good one, this is exactly the reason it been developed by Java core language.

Its true that Android system can destroy and recreate your Fragment. So you can do this:

public MyFragment() {
//  An empty constructor for Android System to use, otherwise exception may occur.
}

public MyFragment(int someInt) {
    Bundle args = new Bundle();
    args.putInt("someInt", someInt);
    setArguments(args);
}

It will allow you to pull someInt from getArguments() latter on, even if the Fragment been recreated by the system. This is more elegant solution than static constructor.

For my opinion static constructors are useless and should not be used. Also they will limit you if in the future you would like to extend this Fragment and add more functionality to the constructor. With static constructor you can't do this.

Update:

Android added inspection that flag all non-default constructors with an error.
I recommend to disable it, for the reasons mentioned above.

Extracting Path from OpenFileDialog path/filename

if (openFileDialog1.ShowDialog(this) == DialogResult.OK)
{
    strfilename = openFileDialog1.InitialDirectory + openFileDialog1.FileName;
}

TypeError : Unhashable type

You are creating a set via set(...) call, and set needs hashable items. You can't have set of lists. Because list's arent hashable.

[[(a,b) for a in range(3)] for b in range(3)] is a list. It's not a hashable type. The __hash__ you saw in dir(...) isn't a method, it's just None.

A list comprehension returns a list, you don't need to explicitly use list there, just use:

>>> [[(a,b) for a in range(3)] for b in range(3)]
[[(0, 0), (1, 0), (2, 0)], [(0, 1), (1, 1), (2, 1)], [(0, 2), (1, 2), (2, 2)]]

Try those:

>>> a = {1, 2, 3}
>>> b= [1, 2, 3]
>>> type(a)
<class 'set'>
>>> type(b)
<class 'list'>
>>> {1, 2, []}
Traceback (most recent call last):
  File "<stdin>", line 1, in <module>
TypeError: unhashable type: 'list'
>>> print([].__hash__)
None
>>> [[],[],[]] #list of lists
[[], [], []]
>>> {[], [], []} #set of lists
Traceback (most recent call last):
  File "<stdin>", line 1, in <module>
TypeError: unhashable type: 'list'

SCCM 2012 application install "Failed" in client Software Center

I'm assuming you figured this out already but:

Technical Reference for Log Files in Configuration Manager

That's a list of client-side logs and what they do. They are located in Windows\CCM\Logs

AppEnforce.log will show you the actual command-line executed and the resulting exit code for each Deployment Type (only for the new style ConfigMgr Applications)

This is my go-to for troubleshooting apps. Haven't really found any other logs that are exceedingly useful.

How do you send an HTTP Get Web Request in Python?

You can use urllib2

import urllib2
content = urllib2.urlopen(some_url).read()
print content

Also you can use httplib

import httplib
conn = httplib.HTTPConnection("www.python.org")
conn.request("HEAD","/index.html")
res = conn.getresponse()
print res.status, res.reason
# Result:
200 OK

or the requests library

import requests
r = requests.get('https://api.github.com/user', auth=('user', 'pass'))
r.status_code
# Result:
200

How to call Stored Procedure in Entity Framework 6 (Code-First)?

public static string ToSqlParamsString(this IDictionary<string, string> dict)
        {
            string result = string.Empty;
            foreach (var kvp in dict)
            {
                result += $"@{kvp.Key}='{kvp.Value}',";
            }
            return result.Trim(',', ' ');
        }

public static List<T> RunSproc<T>(string sprocName, IDictionary<string, string> parameters)
        {
            string command = $"exec {sprocName} {parameters.ToSqlParamsString()}";
            return Context.Database.SqlQuery<T>(command).ToList();
        }

What is the difference between SQL, PL-SQL and T-SQL?

SQL is a standard and there are many database vendors like Microsoft,Oracle who implements this standard using their own proprietary language.

Microsoft uses T-SQL to implement SQL standard to interact with data whereas oracle uses PL/SQL.

How to call a VbScript from a Batch File without opening an additional command prompt

If you want to fix vbs associations type

regsvr32 vbscript.dll
regsvr32 jscript.dll
regsvr32 wshext.dll
regsvr32 wshom.ocx
regsvr32 wshcon.dll
regsvr32 scrrun.dll

Also if you can't use vbs due to management then convert your script to a vb.net program which is designed to be easy, is easy, and takes 5 minutes.

Big difference is functions and subs are both called using brackets rather than just functions.

So the compilers are installed on all computers with .NET installed.

See this article here on how to make a .NET exe. Note the sample is for a scripting host. You can't use this, you have to put your vbs code in as .NET code.

How can I convert a VBScript to an executable (EXE) file?

(change) vs (ngModelChange) in angular

1 - (change) is bound to the HTML onchange event. The documentation about HTML onchange says the following :

Execute a JavaScript when a user changes the selected option of a <select> element

Source : https://www.w3schools.com/jsref/event_onchange.asp

2 - As stated before, (ngModelChange) is bound to the model variable binded to your input.

So, my interpretation is :

  • (change) triggers when the user changes the input
  • (ngModelChange) triggers when the model changes, whether it's consecutive to a user action or not

Add Foreign Key relationship between two Databases

In my experience, the best way to handle this when the primary authoritative source of information for two tables which are related has to be in two separate databases is to sync a copy of the table from the primary location to the secondary location (using T-SQL or SSIS with appropriate error checking - you cannot truncate and repopulate a table while it has a foreign key reference, so there are a few ways to skin the cat on the table updating).

Then add a traditional FK relationship in the second location to the table which is effectively a read-only copy.

You can use a trigger or scheduled job in the primary location to keep the copy updated.

Can't bind to 'ngIf' since it isn't a known property of 'div'

If you are using RC5 then import this:

import { CommonModule } from '@angular/common';  
import { BrowserModule } from '@angular/platform-browser';

and be sure to import CommonModule from the module that is providing your component.

 @NgModule({
    imports: [CommonModule],
    declarations: [MyComponent]
  ...
})
class MyComponentModule {}

How to select clear table contents without destroying the table?

I use this code to remove my data but leave the formulas in the top row. It also removes all rows except for the top row and scrolls the page up to the top.

Sub CleanTheTable()
    Application.ScreenUpdating = False
    Sheets("Data").Select
    ActiveSheet.ListObjects("TestTable").HeaderRowRange.Select
    'Remove the filters if one exists.
    If ActiveSheet.FilterMode Then
    Selection.AutoFilter
    End If
    'Clear all lines but the first one in the table leaving formulas for the next go round.
    With Worksheets("Data").ListObjects("TestTable")
    .Range.AutoFilter
    On Error Resume Next
    .DataBodyRange.Offset(1).Resize(.DataBodyRange.Rows.Count - 1, .DataBodyRange.Columns.Count).Rows.Delete
    .DataBodyRange.Rows(1).SpecialCells(xlCellTypeConstants).ClearContents
    ActiveWindow.SmallScroll Down:=-10000

    End With
Application.ScreenUpdating = True
End Sub

Format XML string to print friendly XML string

I tried:

internal static void IndentedNewWSDLString(string filePath)
{
    var xml = File.ReadAllText(filePath);
    XDocument doc = XDocument.Parse(xml);
    File.WriteAllText(filePath, doc.ToString());
}

it is working fine as expected.

How can I resolve the error: "The command [...] exited with code 1"?

This builds on the answer from JaredPar... and is for VS2017. The same "Build and Run" options are present in Visual Studio 2017.

I was getting, The command "chmod +x """ exited with code 1

In the build output window, I searched for "Error" and found a few errors in the same general area. I was able to click on a link in the build output, and found that the error involved this entry in the .targets file:

  <Target Name="ChmodChromeDriver" BeforeTargets="BeforeBuild" Condition="'$(WebDriverPlatform)' != 'win32'">
    <Exec Command="chmod +x &quot;$(ChromeDriverSrcPath)&quot;" />
  </Target>

In the build output, I also found a more detailed error message that essentially stated that it couldn't find Selenium.WebDriver.ChromeDriver v2.36 in the packages folder it was looking in. I checked the project's NuGet packages, and version 2.36 was indeed in the list of installed packages. I did find the package files for 2.36, and changed the attributes on the folder, subfolders and files from "Read Only" to "Read/Write". Built, but same failure. Sometimes "updating" to a different version of the package and then updating back to the original can fix this type of error. So I "updated" the reference in Manage NuGet packages to 2.37, built, failed, then "updated" back to 2.36, built, and the build succeeded without the "chmod +x" error message.

The project I was building was based on a Visual Studio Project template for Appium test tooling, template name "Develop_Automated_Test".

How to check if a query string value is present via JavaScript?

You could also use a regular expression:

/[?&]q=/.test(location.search)

What does Html.HiddenFor do?

It creates a hidden input on the form for the field (from your model) that you pass it.

It is useful for fields in your Model/ViewModel that you need to persist on the page and have passed back when another call is made but shouldn't be seen by the user.

Consider the following ViewModel class:

public class ViewModel
{
    public string Value { get; set; }
    public int Id { get; set; }
}

Now you want the edit page to store the ID but have it not be seen:

<% using(Html.BeginForm() { %>
    <%= Html.HiddenFor(model.Id) %><br />
    <%= Html.TextBoxFor(model.Value) %>
<% } %>

This results in the equivalent of the following HTML:

<form name="form1">
    <input type="hidden" name="Id">2</input>
    <input type="text" name="Value" value="Some Text" />
</form>

AngularJS : Clear $watch

You can also clear the watch inside the callback if you want to clear it right after something happens. That way your $watch will stay active until used.

Like so...

var clearWatch = $scope.$watch('quartzCrystal', function( crystal ){
  if( isQuartz( crystal )){
    // do something special and then stop watching!
    clearWatch();
  }else{
    // maybe do something special but keep watching!
  } 
}

How to squash commits in git after they have been pushed?

Squash commits locally with

git rebase -i origin/master~4 master

and then force push with

git push origin +master

Difference between --force and +

From the documentation of git push:

Note that --force applies to all the refs that are pushed, hence using it with push.default set to matching or with multiple push destinations configured with remote.*.push may overwrite refs other than the current branch (including local refs that are strictly behind their remote counterpart). To force a push to only one branch, use a + in front of the refspec to push (e.g git push origin +master to force a push to the master branch).

How do you tell if caps lock is on using JavaScript?

In JQuery. This covers the event handling in Firefox and will check for both unexpected uppercase and lowercase characters. This presupposes an <input id="password" type="password" name="whatever"/>element and a separate element with id 'capsLockWarning' that has the warning we want to show (but is hidden otherwise).

$('#password').keypress(function(e) {
    e = e || window.event;

    // An empty field resets the visibility.
    if (this.value === '') {
        $('#capsLockWarning').hide();
        return;
    }

    // We need alphabetic characters to make a match.
    var character = String.fromCharCode(e.keyCode || e.which);
    if (character.toUpperCase() === character.toLowerCase()) {
        return;
    }

    // SHIFT doesn't usually give us a lowercase character. Check for this
    // and for when we get a lowercase character when SHIFT is enabled. 
    if ((e.shiftKey && character.toLowerCase() === character) ||
        (!e.shiftKey && character.toUpperCase() === character)) {
        $('#capsLockWarning').show();
    } else {
        $('#capsLockWarning').hide();
    }
});

How to write to file in Ruby?

This is preferred approach in most cases:

 File.open(yourfile, 'w') { |file| file.write("your text") }

When a block is passed to File.open, the File object will be automatically closed when the block terminates.

If you don't pass a block to File.open, you have to make sure that file is correctly closed and the content was written to file.

begin
  file = File.open("/tmp/some_file", "w")
  file.write("your text") 
rescue IOError => e
  #some error occur, dir not writable etc.
ensure
  file.close unless file.nil?
end

You can find it in documentation:

static VALUE rb_io_s_open(int argc, VALUE *argv, VALUE klass)
{
    VALUE io = rb_class_new_instance(argc, argv, klass);
    if (rb_block_given_p()) {
        return rb_ensure(rb_yield, io, io_close, io);
    }
    return io;
}

Elevating process privilege programmatically?

You should use Impersonation to elevate the state.

WindowsIdentity identity = new WindowsIdentity(accessToken);
WindowsImpersonationContext context = identity.Impersonate();

Don't forget to undo the impersonated context when you are done.

ValueError: max() arg is an empty sequence

When the length of v will be zero, it'll give you the value error.

You should check the length or you should check the list first whether it is none or not.

if list:
    k.index(max(list))

or

len(list)== 0

-bash: export: `=': not a valid identifier

Try to surround the path with quotes, and remove the spaces

export PYTHONPATH="/home/user/my_project":$PYTHONPATH

And don't forget to preserve previous content suffixing by :$PYTHONPATH (which is the value of the variable)

Execute the following command to check everything is configured correctly:

echo $PYTHONPATH

Execution failed app:processDebugResources Android Studio

change the sdk version to 21 if its 20 and also make sure the build version as 21.0.1

What is JavaScript garbage collection?

"In computer science, garbage collection (GC) is a form of automatic memory management. The garbage collector, or just collector, attempts to reclaim garbage, or memory used by objects that will never be accessed or mutated again by the application."

All JavaScript engines have their own garbage collectors, and they may differ. Most time you do not have to deal with them because they just do what they supposed to do.

Writing better code mostly depends of how good do you know programming principles, language and particular implementation.

In Bash, how to add "Are you sure [Y/n]" to any command or alias?

Below code is combining two things

  1. shopt -s nocasematch that will take care of case insensitive

  2. and if condition that will accept both the input either you pass yes,Yes,YES,y.

    shopt -s nocasematch

    if [[ sed-4.2.2.$LINE =~ (yes|y)$ ]]

    then exit 0

    fi

Python Pandas - Missing required dependencies ['numpy'] 1

First, try to import numpy on it's own, like so:

import numpy as np

I got this message:

ImportError: Something is wrong with the numpy installation. While importing 
we detected an older version of numpy in 
['/home/michael/.local/lib/python3.6/site-packages/numpy']. One method of 
fixing this is to repeatedly uninstall numpy until none is found, then 
reinstall this version.

So do what it says, keep uninstalling numpy until there is none, and then reinstall.

This worked for me.

How to convert string to char array in C++?

Ok, i am shocked that no one really gave a good answer, now my turn. There are two cases;

  1. A constant char array is good enough for you so you go with,

    const char *array = tmp.c_str();
    
  2. Or you need to modify the char array so constant is not ok, then just go with this

    char *array = &tmp[0];
    

Both of them are just assignment operations and most of the time that is just what you need, if you really need a new copy then follow other fellows answers.

what is Ljava.lang.String;@

According to the Java Virtual Machine Specification (Java SE 8), JVM §4.3.2. Field Descriptors:

FieldType term | Type      | Interpretation
-------------- | --------- | --------------
L ClassName ;  | reference | an instance of class ClassName
[              | reference | one array dimension
...            | ...       | ...

the expression [Ljava.lang.String;@45a877 means this is an array ( [ ) of class java.lang.String ( Ljava.lang.String; ). And @45a877 is the address where the String object is stored in memory.

Call method when home button pressed

I found that when I press the button HOME the onStop() method is called.You can use the following piece of code to monitor it:

@Override
    protected void onStop() 
    {
        super.onStop();
        Log.d(tag, "MYonStop is called");
        // insert here your instructions
    }

Cannot get a text value from a numeric cell “Poi”

use the code
cell.setCellType(Cell.CELL_TYPE_STRING);
before reading the string value, Which can help you.
I am using POI version 3.17 Beta1 version, sure the version compatibility also..

MaxLength Attribute not generating client-side validation attributes

StringLength works great, i used it this way:

[StringLength(25,MinimumLength=1,ErrorMessage="Sorry only 25 characters allowed for 
              ProductName")]
public string ProductName { get; set; }

or Just Use RegularExpression without StringLength:

[RegularExpression(@"^[a-zA-Z0-9'@&#.\s]{1,25}$", ErrorMessage = "Reg Says Sorry only 25 
                   characters allowed for ProductName")]    
public string ProductName { get; set; }

but for me above methods gave error in display view, cause i had already ProductName field in database which had more than 25 characters

so finally i came across this and this post and tried to validate without model like this:

 <div class="editor-field">
 @Html.TextBoxFor(model => model.ProductName, new
 {
 @class = "form-control",
 data_val = "true",
 data_val_length = "Sorry only 25 characters allowed for ProductName",
 data_val_length_max = "25",
 data_val_length_min = "1"
 })
 <span class="validation"> @Html.ValidationMessageFor(model => model.ProductName)</span>
 </div>

this solved my issue, you can also do validation manually using jquery or using ModelState.AddModelError

hope helps someone.

illegal use of break statement; javascript

You need to make sure requestAnimFrame stops being called once game == 1. A break statement only exits a traditional loop (e.g. while()).

function loop() {
    if (isPlaying) {
        jet1.draw();
        drawAllEnemies();
        if (game != 1) {
            requestAnimFrame(loop);
        }
    }
}

Or alternatively you could simply skip the second if condition and change the first condition to if (isPlaying && game !== 1). You would have to make a variable called game and give it a value of 0. Add 1 to it every game.

how to include js file in php?

I have never been a fan of closing blocks of PHP to output content to the browser, I prefer to have my output captured so if at some point within my logic I decide I want to change my output (after output has already been sent) I can just delete the current buffer.

But as Pekka said, the main reason you are having issues with your javascript inclusion is because your using href to specify the location of the js file where as you should be using src.

If you have a functions file with your functions inside then add something like:

function js_link($src)
{
    if(file_exists("my/html/root/" . $src))
    {
        //we know it will exists within the HTTP Context
        return sprintf("<script type=\"text/javascript\" src=\"%s\"></script>",$src);
    }
    return "<!-- Unable to load " . $src . "-->";
}

The n in your code without the need for closing your blocks with ?> you can just use:

echo js_link("jquery/1.6/main.js");

Specify path to node_modules in package.json

In short: It is not possible, and as it seems won't ever be supported (see here https://github.com/npm/npm/issues/775).

There are some hacky work-arrounds with using the CLI or ENV-Variables (see the current selected answer), .npmrc-Config-Files or npm link - what they all have in common: They are never just project-specific, but always some kind of global Solutions.

For me, none of those solutions are really clean because contributors to your project always need to create some special configuration or have some special knowledge - they can't just npm install and it works.

So: Either you will have to put your package.json in the same directory where you want your node_modules installed, or live with the fact that they will always be in the root-dir of your project.

How to start automatic download of a file in Internet Explorer?

I used this, seems working and is just simple JS, no framework:

Your file should start downloading in a few seconds. 
If downloading doesn't start automatically
<a id="downloadLink" href="[link to your file]">click here to get your file</a>.

<script> 
    var downloadTimeout = setTimeout(function () {
        window.location = document.getElementById('downloadLink').href;
    }, 2000);
</script>

NOTE: this starts the timeout in the moment the page is loaded.

How to extract week number in sql

After converting your varchar2 date to a true date datatype, then convert back to varchar2 with the desired mask:

to_char(to_date('01/02/2012','MM/DD/YYYY'),'WW')

If you want the week number in a number datatype, you can wrap the statement in to_number():

to_number(to_char(to_date('01/02/2012','MM/DD/YYYY'),'WW'))

However, you have several week number options to consider:

WW  Week of year (1-53) where week 1 starts on the first day of the year and continues to the seventh day of the year.
W   Week of month (1-5) where week 1 starts on the first day of the month and ends on the seventh.
IW  Week of year (1-52 or 1-53) based on the ISO standard.

How do I remove javascript validation from my eclipse project?

In addition, if you are using Tern eclipse IDE or IBM Node.js Tools for Eclipse, you may need to disable JSHint and other libraries that you don't want.

To disable this, Project Properties > Tern > Modules > JSHint or any other library that you don't want. enter image description here

Validation failed for one or more entities. See 'EntityValidationErrors' property for more details

Here's how you can check the contents of the EntityValidationErrors in Visual Studio (without writing any extra code) i.e. during Debugging in the IDE.

The Problem?

You are right, the Visual Studio debugger's View Details Popup doesn't show the actual errors inside the EntityValidationErrors collection .

enter image description here

The Solution!

Just add the following expression in a Quick Watch window and click Reevaluate.

((System.Data.Entity.Validation.DbEntityValidationException)$exception).EntityValidationErrors

In my case, see how I am able to expand into the ValidationErrors List inside the EntityValidationErrors collection

enter image description here

References: mattrandle.me blog post, @yoel's answer

How to semantically add heading to a list

Try defining a new class, ulheader, in css. p.ulheader ~ ul selects all that immediately follows My Header

p.ulheader ~ ul {
   margin-top:0;
{
p.ulheader {
   margin-bottom;0;
}

Is there a bash command which counts files?

This simple one-liner should work in any shell, not just bash:

ls -1q log* | wc -l

ls -1q will give you one line per file, even if they contain whitespace or special characters such as newlines.

The output is piped to wc -l, which counts the number of lines.

Is there a way to call a stored procedure with Dapper?

Same from above, bit more detailed

Using .Net Core

Controller

public class TestController : Controller
{
    private string connectionString;

    public IDbConnection Connection
    {
        get { return new SqlConnection(connectionString); }
    }

    public TestController()
    {
        connectionString = @"Data Source=OCIUZWORKSPC;Initial Catalog=SocialStoriesDB;Integrated Security=True";
    }

    public JsonResult GetEventCategory(string q)
    {
        using (IDbConnection dbConnection = Connection)
        {
            var categories = dbConnection.Query<ResultTokenInput>("GetEventCategories", new { keyword = q },
    commandType: CommandType.StoredProcedure).FirstOrDefault();

            return Json(categories);
        }
    }

    public class ResultTokenInput
    {
        public int ID { get; set; }
        public string name { get; set; }            
    }
}

Stored Procedure ( parent child relation )

create PROCEDURE GetEventCategories
@keyword as nvarchar(100)
AS
    BEGIN

    WITH CTE(Id, Name, IdHierarchy,parentId) AS
    (
      SELECT 
        e.EventCategoryID as Id, cast(e.Title as varchar(max)) as Name,
        cast(cast(e.EventCategoryID as char(5)) as varchar(max)) IdHierarchy,ParentID
      FROM 
        EventCategory e  where e.Title like '%'+@keyword+'%'
     -- WHERE 
      --  parentid = @parentid

      UNION ALL

      SELECT 
        p.EventCategoryID as Id, cast(p.Title + '>>' + c.name as varchar(max)) as Name,
        c.IdHierarchy + cast(p.EventCategoryID as char(5)),p.ParentID
      FROM 
        EventCategory p 
      JOIN  CTE c ON c.Id = p.parentid

        where p.Title like '%'+@keyword+'%'
    )
    SELECT 
      * 
    FROM 
      CTE
    ORDER BY 
      IdHierarchy

References in case

using System;
using System.Collections.Generic;
using System.Linq;
using System.Threading.Tasks;
using Microsoft.AspNetCore.Mvc;
using Microsoft.AspNetCore.Http;
using SocialStoriesCore.Data;
using Microsoft.EntityFrameworkCore;
using Dapper;
using System.Data;
using System.Data.SqlClient;

Setup a Git server with msysgit on Windows

I am not sure why anyone hasn't suggested http://gitblit.com. Pure java based solution, allow HTTP protocol and really easy to setup.

How do I put a border around an Android textview?

You can create custom background for your text view. Steps 1. Go to your project. 2. Go to resources and right click to drawable. 3. Click on New -> Drawable Resource File 4. Give name to you file 5. Paste following code in the file

<shape xmlns:android="http://schemas.android.com/apk/res/android">
<stroke android:width="1dp" android:color="@color/colorBlack" />
<padding android:left="1dp"
    android:top="1dp"
    android:right="1dp"
    android:bottom="1dp" />
<corners android:radius="6dp" />
<solid android:color="#ffffffff" />

  1. For your text view where you want to use it as backgroud,

    android:background="@drawable/your_fileName"

How do you read scanf until EOF in C?

Try:

while(scanf("%15s", words) != EOF)

You need to compare scanf output with EOF

Since you are specifying a width of 15 in the format string, you'll read at most 15 char. So the words char array should be of size 16 ( 15 +1 for null char). So declare it as:

char words[16];

Play audio from a stream using C#

Bass can do just this. Play from Byte[] in memory or a through file delegates where you return the data, so with that you can play as soon as you have enough data to start the playback..

Copy all values from fields in one class to another through reflection

My solution:

public static <T > void copyAllFields(T to, T from) {
        Class<T> clazz = (Class<T>) from.getClass();
        // OR:
        // Class<T> clazz = (Class<T>) to.getClass();
        List<Field> fields = getAllModelFields(clazz);

        if (fields != null) {
            for (Field field : fields) {
                try {
                    field.setAccessible(true);
                    field.set(to,field.get(from));
                } catch (IllegalAccessException e) {
                    e.printStackTrace();
                }
            }
        }
    }

public static List<Field> getAllModelFields(Class aClass) {
    List<Field> fields = new ArrayList<>();
    do {
        Collections.addAll(fields, aClass.getDeclaredFields());
        aClass = aClass.getSuperclass();
    } while (aClass != null);
    return fields;
}

Console app arguments, how arguments are passed to Main method

in visual studio you can also do like that to pass simply or avoiding from comandline argument

 static void Main(string[] args)
    {
        if (args == null)
        {
            Console.WriteLine("args is null"); // Check for null array
        }
        else
        {
            args=new string[2];
            args[0] = "welcome in";
            args[1] = "www.overflow.com";
            Console.Write("args length is ");
            Console.WriteLine(args.Length); // Write array length
            for (int i = 0; i < args.Length; i++) // Loop through array
            {
                string argument = args[i];
                Console.Write("args index ");
                Console.Write(i); // Write index
                Console.Write(" is [");
                Console.Write(argument); // Write string
                Console.WriteLine("]");
            }
        }